1 Introduction and basic assumptions

This work deals with existence of solutions for strongly nonlinear boundary value problem whose model is:

$$\begin{aligned} \left\{ \begin{array}{c} A(u)-{\text {div}}\Phi (u)+g(x, u, \nabla u)=f \quad in \quad \Omega \\ \quad \quad \quad \quad \quad \quad \quad \quad \quad \quad \quad \quad \quad u \equiv 0, \quad on \quad \partial \Omega \end{array}\right. \end{aligned}$$
(1.1)

where \(\Omega \) be a bounded domain of \({\mathbb {R}}^{N}, N \ge 2 ,\) \( A(u)=-{\text {div}} a(x, u, \nabla u) \) be a Leray-Lions operator defined from the space \(W_{0}^{1} L_{\varphi }(\Omega )\) into its dual \(W^{-1} L_{\psi }(\Omega ),\) and \(\Phi \in \mathrm {C}^{0}\left( \mathrm {R}, \mathrm {R}^{N}\right) \) . where a is a function satisfying the following conditions :

$$\begin{aligned} a(x, s, \xi ): \Omega \times {\mathbb {R}} \times {\mathbb {R}}^{N} \longrightarrow {\mathbb {R}}^{N} \text {is a Carath}\acute{\mathrm{e}}\text {odory function}. \end{aligned}$$
(1.2)

There exist two Musielak–Orlicz functions \(\varphi \) and \(\gamma \) such that \(\gamma \prec \prec \varphi ,\) a positive function \(d(\cdot ) \in E_{\psi }(\Omega )\) and positive constants \(k_{1},k_{2}\text { and } k_{3}\) such that for a.e. \(x \in \Omega \) and for all \(s \in {\mathbb {R}}, \xi \in {\mathbb {R}}^{N}\)

$$\begin{aligned}&|a(x, s, \xi )| \le k_{1}\left( d(x)+\psi _{x}^{-1} \gamma (x, k_{2}|s|)\right) +\psi _{x}^{-1} \varphi (x, k_{3}|\xi |); \end{aligned}$$
(1.3)
$$\begin{aligned}&\left( a(x, s, \xi )-a\left( x, s, \xi ^{\prime }\right) \right) \left( \xi -\xi ^{\prime }\right) >0; \end{aligned}$$
(1.4)
$$\begin{aligned}&a(x, s, \xi ) . \xi \ge \alpha \varphi (x,|\xi |). \end{aligned}$$
(1.5)

Furthermore, let \(g(x, s, \xi ): \Omega \times {\mathbb {R}} \times {\mathbb {R}}^{N} \longrightarrow {\mathbb {R}}\) be a Carathéodory function such that for a.e. \(x \in \Omega \) and for all \(s \in {\mathbb {R}}, \xi \in {\mathbb {R}}^{N},\) satisfying the following conditions

$$\begin{aligned}&|g(x, s, \xi )| \le c(x)+b(|s|) \varphi (x,|\xi |); \end{aligned}$$
(1.6)
$$\begin{aligned}&g(x, s, \xi ) s \ge 0; \end{aligned}$$
(1.7)

where \(b: {\mathbb {R}}^{+} \longrightarrow {\mathbb {R}}^{+}\) is a continuous positive function which belongs to \(L^{1}\left( {\mathbb {R}}^{+}\right) \) and \(c(\cdot ) \in L^{1}(\Omega )\) The right-hand side of (1.1) and \(\Phi : {\mathbb {R}} \rightarrow {\mathbb {R}}^{N}\) are assumed to satisfy

$$\begin{aligned}&f \in W^{-1} E_{\psi }(\Omega ); \end{aligned}$$
(1.8)
$$\begin{aligned}&\Phi \in {\mathcal {C}}^{0}\left( {\mathbb {R}}, {\mathbb {R}}^{N}\right) . \end{aligned}$$
(1.9)

Note that no growth hypothesis is assumed on the function \(\Phi \), which implies that the term \( -\text{ div } \Phi (u) \) may be meaningless, even as a distribution.

Several researches deals with the existence solutions of elliptic and parabolic problems under various assumptions and in different contexts (see [1,2,3,4,5,6,7,8,9,10, 13,14,15,16,17,18,19,20, 24,25,28, 35, 37, 39, 40] for more details), indeed we can’t recite all examples; we will just choose some of them, So we mention that:

the problem (1.1) was treated by Boccardo (see [23]) in the case \( g\equiv 0 \) and for p such that \(2-1 / N<p<N\) where he proved the existence and regularity of an entropy solution u that is \(u \in W_{0}^{1, q}(\Omega ), \quad q<{\tilde{p}}=\frac{(p-1) N}{N-1}, \) \(T_{k}(u) \in W_{0}^{1, p}(\Omega ), \quad \forall k>0.\) The same problem have been studied by Diperna and lions in [26] where they introduced the idea of renormalized solutions.

In the framework of variable exponent Sobolev spaces in [12] have proved the existence result of solutions for the problem 1.1 without sign condition involving nonstandard growth.

In the setting of Musielak spaces and in variational case, the existence of a weak solution for the problem (1.1) was treated by Ahmed Oubeid, Benkirane and Sidi El Vally in [11] where \( {\text {div}}\Phi \equiv 0.\)

Our purpose in this paper is to show the existence of renormalized solutions for problem (1.1) in Musielak Orlicz spaces in the case where the Musielak–Orlicz function \( \varphi \) doesn’t satisfy the \(\Delta _{2}\) condition,while the right-hand side belongs to \(W^{-1} E_{\psi }(\Omega )\), \(\Phi \in {\mathcal {C}}^{0}\left( {\mathbb {R}}, {\mathbb {R}}^{N}\right) .\) and a nonlinearity \(g(x, s, \xi )\) having natural growth with respect to the gradient.

The paper is organized as follows: In Sect. 2 , we give some preliminaries and background. Section 3 is devoted to some technical lemmas which can be used to our result. In the final Sect. 4, we state our main result and give the prove of an existence solution.

2 Some preliminaries and background

Here we give some definitions and properties that concern Musielak–Orlicz spaces (see [34]).

Let \(\Omega \) be an open subset of \({\mathbb {R}}^{n}\), a Musielak–Orlicz function \( \varphi \) is a real-valued function defined in \(\Omega \times {\mathbb {R}}_{+}\) such that

  1. (a)

    \( \varphi (x, t)\) is an N-function i.e. convex, nondecreasing, continuous, \(\varphi (x, 0)=0,\) \( \varphi (x, t)>0\) for all \(t>0\) and

    $$\begin{aligned} \begin{aligned} \lim _{t \rightarrow 0} \sup _{x \in \Omega } \frac{\varphi (x, t)}{t}&=0 ,\qquad \lim _{t \rightarrow \infty } \inf _{x \in \Omega } \frac{\varphi (x, t)}{t}&=0 \end{aligned} \end{aligned}$$
  2. (b)

    \(\varphi (x, t)\) is a measurable function for all \(t \ge 0\) .

    Now, let \(\varphi _{x}(t)=\varphi (x, t)\) and let \(\varphi _{x}^{-1}\) be the non-negative reciprocal function with respect to t,  i.e the function that satisfies

    $$\begin{aligned} \varphi _{x}^{-1}(\varphi (x, t))=\varphi \left( x, \varphi _{x}^{-1}(t)\right) =t \end{aligned}$$

The Musielak–Orlicz function \(\varphi \) is said to satisfy the \(\Delta _{2}\) -condition if for some \(k>0,\) and a non negative function h,  integrable in \(\Omega ,\) we have

$$\begin{aligned} \varphi (x, 2 t) \le k \varphi (x, t)+h(x) \text{ for } \text{ all } x \in \Omega \text{ and } t \ge 0. \end{aligned}$$
(2.1)

When (2.1) holds only for \(t \ge t_{0}>0,\) then \(\varphi \) is said to satisfy the \(\Delta _{2}\) -condition near infinity. Let \( \varphi \) and \(\gamma \) be two Musielak–Orlicz functions, we say that \(\varphi \) dominate \(\gamma \) and we write \(\gamma \prec \varphi ,\) near infinity (resp. globally) if there exist two positive constants c and \(t_{0}\) such that for almost all \(x \in \Omega \)

\(\gamma (x, t) \le \varphi (x, c t)\) for all \(t \ge t_{0}, \quad \left( \text{ resp. } \text{ for } \text{ all } t \ge 0 \text{ i.e. } t_{0}=0\right) \) We say that \(\gamma \) grows essentially less rapidly than \(\varphi \) at 0 (resp. near infinity) and we write \(\gamma \prec \prec \varphi \) if for every positive constant c we have

$$\begin{aligned} \lim _{t \rightarrow 0}\left( \sup _{x \in \Omega } \frac{\gamma (x, c t)}{\varphi (x, t)}\right) =0, \quad \left( \text{ resp. } \lim _{t \rightarrow \infty }\left( \sup _{x \in \Omega } \frac{\gamma (x, c t)}{\varphi (x, t)}\right) =0\right) \end{aligned}$$

Remark 2.1

(see [29]) If \(\gamma \prec \varphi \) near infinity such that \(\gamma \) is locally integrable on \(\Omega ,\) then \(\forall c>0\) there exists a nonnegative integrable function h such that

$$\begin{aligned} \gamma (x, t) \le \varphi (x, c t)+h(x), \text{ for } \text{ all } t \ge 0 \text{ and } \text{ for } \text{ a. } \text{ e. } x \in \Omega . \end{aligned}$$

For a Musielak–Orlicz function \(\varphi \) and a measurable function \(u: \Omega \longrightarrow {\mathbb {R}},\) we define the functional

$$\begin{aligned} \rho _{\varphi , \Omega }(u)=\int _{\Omega } \varphi (x,|u(x)|) d x \end{aligned}$$

The set \(K_{\varphi }(\Omega )=\left\{ u: \Omega \longrightarrow {\mathbb {R}} \text{ measurable } / \rho _{\varphi , \Omega }(u)<\infty \right\} \) is called the Musielak–Orlicz class (or generalized Orlicz class). The Musielak–Orlicz space (the generalized Orlicz spaces) \( L_{\varphi }(\Omega )\) is the vector space generated by \(K_{\varphi }(\Omega ),\) that is, \(L_{\varphi }(\Omega )\) is the smallest linear space containing the set \(K_{\varphi }(\Omega ) .\) Equivalently

$$\begin{aligned} L_{\varphi }(\Omega )=\left\{ u: \Omega \longrightarrow {\mathbb {R}} \text{ measurable } / \rho _{\varphi , \Omega }\left( \frac{u}{\lambda }\right) <\infty , \text{ for } \text{ some } \lambda >0\right\} \end{aligned}$$

For a Musielak–Orlicz function \(\varphi \) we put: \(\psi (x, s)=\sup _{t>0}\{s t-\varphi (x, t)\}, \psi \) is the Musielak–Orlicz function complementary to \(\varphi \) (or conjugate of \(\varphi \) ) in the sens of Young with respect to the variable s In the space \(L_{\varphi }(\Omega )\) we define the following two norms:

$$\begin{aligned} \Vert u\Vert _{\varphi , \Omega }=\inf \left\{ \lambda >0 / \int _{\Omega } \varphi \left( x, \frac{|u(x)|}{\lambda }\right) d x \le 1\right\} \end{aligned}$$

which is called the Luxemburg norm and the so-called Orlicz norm by:

$$\begin{aligned} \Vert |u|\Vert _{\varphi , \Omega }=\sup _{\Vert v\Vert _{\psi } \le 1} \int _{\Omega }|u(x) v(x)| d x \end{aligned}$$

where \(\psi \) is the Musielak Orlicz function complementary to \(\varphi .\) These two norms are equivalent (see [34])

The closure in \(L_{\varphi }(\Omega )\) of the bounded measurable functions with compact support in \({\bar{\Omega }}\) is denoted by \(E_{\varphi }(\Omega )\), It is a separable space (see [34], Theorem 7.10) .

We say that sequence of functions \(u_{n} \in L_{\varphi }(\Omega )\) is modular convergent to \(u \in \) \(L_{\varphi }(\Omega )\) if there exists a constant \(\lambda >0\) such that

$$\begin{aligned} \lim _{n \rightarrow \infty } \rho _{\varphi , \Omega }\left( \frac{u_{n}-u}{\lambda }\right) =0. \end{aligned}$$

For any fixed nonnegative integer m we define

$$\begin{aligned} W^{m} L_{\varphi }(\Omega )=\left\{ u \in L_{\varphi }(\Omega ): \forall |\alpha | \le m, D^{\alpha } u \in L_{\varphi }(\Omega )\right\} \end{aligned}$$

and

$$\begin{aligned} W^{m} E_{\varphi }(\Omega )=\left\{ u \in E_{\varphi }(\Omega ): \forall |\alpha | \le m, D^{\alpha } u \in E_{\varphi }(\Omega )\right\} \end{aligned}$$

where \(\alpha =\left( \alpha _{1}, \ldots , \alpha _{n}\right) \) with nonnegative integers \(\alpha _{i},|\alpha |=\left| \alpha _{1}\right| +\ldots +\left| \alpha _{n}\right| \) and \(D^{\alpha } u\) denote the distributional derivatives. The space \(W^{m} L_{\varphi }(\Omega )\) is called the Musielak Orlicz Sobolev space.

Let

$$\begin{aligned} {\bar{\rho }}_{\varphi , \Omega }(u)=\sum _{|\alpha | \le m} \rho _{\varphi , \Omega }\left( D^{\alpha } u\right) \text{ and } \Vert u\Vert _{\varphi , \Omega }^{m}=\inf \left\{ \lambda >0: {\bar{\rho }}_{\varphi , \Omega }\left( \frac{u}{\lambda }\right) \le 1\right\} \end{aligned}$$

for \(u \in W^{m} L_{\varphi }(\Omega ) .\) These functionals are a convex modular and a norm on \(W^{m} L_{\varphi }(\Omega ),\) respectively, and the pair \(\left( W^{m} L_{\varphi }(\Omega ),\Vert \Vert _{\varphi , \Omega }^{m}\right) \) is a Banach space if \(\varphi \) satisfies the following condition (see [34]):

$$\begin{aligned} \text{ there } \text{ exist } \text{ a } \text{ constant } c_{0}>0 \text{ such } \text{ that } \inf _{x \in \Omega } \varphi (x, 1) \ge c_{0} \end{aligned}$$
(2.2)

The space \(W^{m} L_{\varphi }(\Omega )\) will always be identified to a subspace of the product \(\prod _{|\alpha | \le m} L_{\varphi }(\Omega )=\Pi L_{\varphi },\) this subspace is \(\sigma \left( \Pi L_{\varphi }, \Pi E_{\psi }\right) \) closed.

The space \(W_{0}^{m} L_{\varphi }(\Omega )\) is defined as the \(\sigma \left( \Pi L_{\varphi }, \Pi E_{\psi }\right) \) closure of \({\mathcal {D}}(\Omega )\) in \(W^{m} L_{\varphi }(\Omega ) .\) and the space \(W_{0}^{m} E_{\varphi }(\Omega )\) as the (norm) closure of the Schwartz space \({\mathcal {D}}(\Omega )\) in \(W^{m} L_{\varphi }(\Omega ).\)

Let \(W_{0}^{m} L_{\varphi }(\Omega )\) be the \(\sigma \left( \Pi L_{\varphi }, \Pi E_{\psi }\right) \) closure of \({\mathcal {D}}(\Omega )\) in \(W^{m} L_{\varphi }(\Omega )\) The following spaces of distributions will also be used:

$$\begin{aligned} W^{-m} L_{\psi }(\Omega )=\left\{ f \in D^{\prime }(\Omega ) ; f=\sum _{|\alpha | \le m}(-1)^{|\alpha |} D^{\alpha } f_{\alpha } \text{ with } f_{\alpha } \in L_{\psi }(\Omega )\right\} \end{aligned}$$

and

$$\begin{aligned} W^{-m} E_{\psi }(\Omega )=\left\{ f \in D^{\prime }(\Omega ) ; f=\sum _{|\alpha | \le m}(-1)^{|\alpha |} D^{\alpha } f_{\alpha } \text{ with } f_{\alpha } \in E_{\psi }(\Omega )\right\} \end{aligned}$$

We say that a sequence of functions \(u_{n} \in W^{m} L_{\varphi }(\Omega )\) is modular convergent to \(u \in W^{m} L_{\varphi }(\Omega )\) if there exists a constant \(k>0\) such that

$$\begin{aligned} \lim _{n \rightarrow \infty } {\bar{\rho }}_{\varphi , \Omega }\left( \frac{u_{n}-u}{k}\right) =0 \end{aligned}$$

For \(\varphi \) and her complementary function \(\psi ,\) the following inequality is called the Young inequality (see [34]):

$$\begin{aligned} t s \le \varphi (x, t)+\psi (x, s), \quad \forall t, s \ge 0, x \in \Omega \end{aligned}$$
(2.3)

This inequality implies that

$$\begin{aligned} \Vert u\Vert _{\varphi , \Omega } \le \rho _{\varphi , \Omega }(u)+1 \end{aligned}$$
(2.4)

In \(L_{\varphi }(\Omega )\) we have the relation between the norm and the modular

$$\begin{aligned}&{\Vert u\Vert _{\varphi , \Omega } \le \rho _{\varphi , \Omega }(u) \text{ if } \Vert u\Vert _{\varphi , \Omega }>1} \end{aligned}$$
(2.5)
$$\begin{aligned}&{\Vert u\Vert _{\varphi , \Omega } \ge \rho _{\varphi , \Omega }(u) \text{ if } \Vert u\Vert _{\varphi , \Omega } \le 1} \end{aligned}$$
(2.6)

For two complementary Musielak Orlicz functions \(\varphi \) and \(\psi ,\) let \(u \in L_{\varphi }(\Omega )\) and \(v \in L_{\psi }(\Omega ),\) then we have the Holder inequality (see [34]):

$$\begin{aligned} \displaystyle {\left| \int _{\Omega } u(x) v(x) d x\right| \le \Vert u\Vert _{\varphi , \Omega }\Vert |v|\Vert _{\psi , \Omega }} \end{aligned}$$
(2.7)

We will use the following technical lemmas.

3 Some technical lemmas

Lemma 3.1

[19] Let \(\Omega \) be a bounded Lipschitz domain in \({\mathbb {R}}^{N}\) and let \(\varphi \) and \(\psi \) be two complementary Musielak–Orlicz functions which satisfy the following conditions:

  1. (i)

    There exist a constant \(c>0\) such that inf \(_{x \in \Omega } \varphi (x, 1) \ge c\).

  2. (ii)

    There exist a constant \(A>0\) such that for all \(x, y \in \Omega \) with \(|x-y| \le \frac{1}{2}\) we have

    $$\begin{aligned} \frac{\varphi (x, t)}{\varphi (y, t)} \le t^{\left( \frac{A}{\log \left( \frac{1}{| x-y |}\right) }\right) }, \quad \forall t \ge 1 \end{aligned}$$
    (3.1)
  3. (iii)
    $$\begin{aligned} \text{ If } D \subset \Omega \text{ is } \text{ a } \text{ bounded } \text{ measurable } \text{ set, } \text{ then } \displaystyle {\int _{D} \varphi (x, 1) d x<\infty } \end{aligned}$$
    (3.2)
  4. (iv)

    There exist a constant \(C>0\) such that \(\psi (x, 1) \le C \) a.e in \(\Omega \).

Under this assumptions, \({\mathcal {D}}(\Omega )\) is dense in \(L_{\varphi }(\Omega )\) with respect to the modular topology, \({\mathcal {D}}(\Omega )\) is dense in \(W_{0}^{1} L_{\varphi }(\Omega )\) for the modular convergence and \({\mathcal {D}}({\bar{\Omega }})\) is dense in \(W^{1} L_{\varphi }(\Omega )\) the modular convergence.

Consequently, the action of a distribution S in \(W^{-1} L_{\psi }(\Omega )\) on an element u of \(W_{0}^{1} L_{\varphi }(\Omega )\) is well defined. It will be denoted by \(<S, u>\).

Lemma 3.2

[36] Let \(F: {\mathbb {R}} \longrightarrow {\mathbb {R}}\) be uniformly Lipschitzian, with \(F(0)=0 .\) Let \(\varphi \) be a Musielak–Orlicz function and let \(u \in W_{0}^{1} L_{\varphi }(\Omega ) .\) Then \(F(u) \in W_{0}^{1} L_{\varphi }(\Omega )\) Moreover, if the set D of discontinuity points of \(F^{\prime }\) is finite, we have

$$\begin{aligned} \frac{\partial }{\partial x_{i}} F(u)=\left\{ \begin{array}{cc} {F^{\prime }(u) \frac{\partial u}{\partial x_{i}}\qquad \text{ a.e } i n\{x \in \Omega : u(x) \in D\}} \\ 0 \qquad { \text{ a.e } \text{ in } \{x \in \Omega : u(x) \notin D\}} \end{array}\right. \end{aligned}$$

Lemma 3.3

[29] (Poincare’s inequality) Let \(\varphi \) a Musielak Orlicz function which satisfies the assumptions of lemma 3.1, suppose that \(\varphi (x, t)\) decreases with respect of one of coordinate of x Then, that exists a constant \(c>0\) depends only of \( \Omega \) such that

$$\begin{aligned} \int _{\Omega } \varphi (x,|u(x)|) d x \le \int _{\Omega } \varphi (x, c|\nabla u(x)|) d x, \quad \forall u \in W_{0}^{1} L_{\varphi }(\Omega ) \end{aligned}$$

Lemma 3.4

[19] Suppose that \(\Omega \) satisfies the segment property and let \(u \in \) \(W_{0}^{1} L_{\varphi }(\Omega ) .\) Then, there exists a sequence \(\left( u_{n}\right) \subset {\mathcal {D}}(\Omega )\) such that

$$\begin{aligned} u_{n} \rightarrow u \text{ for } \text{ modular } \text{ convergence } \text{ in } W_{0}^{1} L_{\varphi }(\Omega ) \end{aligned}$$

Furthermore, if \(u \in W_{0}^{1} L_{\varphi }(\Omega ) \cap L^{\infty }(\Omega )\) then \(\left\| u_{n}\right\| _{\infty } \le (N+1)\Vert u\Vert _{\infty }\).

Lemma 3.5

Let \(\left( f_{n}\right) , f \in L^{1}(\Omega )\) such that

  1. (i)

    \(f_{n} \ge 0\) a.e in \(\Omega \)

  2. (ii)

    \(f_{n} \longrightarrow f\) a.e in \(\Omega \)

  3. (iii)

    \(\int _{\Omega } f_{n}(x) d x \longrightarrow \int _{\Omega } f(x) d x\) then \(f_{n} \longrightarrow f\) strongly in \(L^{1}(\Omega )\)

Lemma 3.6

[20] If a sequence \(g_{n} \in L_{\varphi }(\Omega )\) converges in measure to a measurable function g and if \(g_{n}\) remains bounded in \(L_{\varphi }(\Omega ),\) then \(g \in L_{\varphi }(\Omega )\) and \( g_{n} \rightharpoonup g \) for \(\sigma \left( \Pi L_{\varphi }, \Pi E_{\psi }\right) \)

Lemma 3.7

(Jensen inequality) [38] Let \(\varphi : {\mathbb {R}} \longrightarrow {\mathbb {R}}\) a convex function and g \(: \Omega \longrightarrow {\mathbb {R}}\) is function measurable, then

$$\begin{aligned} \varphi \left( \int _{\Omega } g d \mu \right) \le \int _{\Omega } \varphi \circ g d \mu . \end{aligned}$$

Lemma 3.8

(The Nemytskii Operator) [29] Let \(\Omega \) be an open subset of \({\mathbb {R}}^{N}\) with finite measure and let \(\varphi \) and \(\psi \) be two Musielak Orlicz functions. Let \(f: \Omega \times {\mathbb {R}}^{p} \longrightarrow {\mathbb {R}}^{q}\) be a Carathodory function such that for a.e. \(x \in \Omega \) and all \(s \in {\mathbb {R}}^{p}:\)

$$\begin{aligned} |f(x, s)| \le c(x)+k_{1} \psi _{x}^{-1} \varphi \left( x, k_{2}|s|\right) \end{aligned}$$

where \(k_{1}\) and \(k_{2}\) are real positives constants and \(c(.) \in E_{\psi }(\Omega )\) Then the Nemytskii Operator \(N_{f}\) defined by \(N_{f}(u)(x)=f(x, u(x))\) is continuous from

$$\begin{aligned} {\mathcal {P}}\left( E_{M}(\Omega ), \frac{1}{k_{2}}\right) ^{p}=\prod \left\{ u \in L_{M}(\Omega ): d\left( u, E_{M}(\Omega )\right) <\frac{1}{k_{2}}\right\} \end{aligned}$$

into \(\left( L_{\psi }(\Omega )\right) ^{q}\) for the modular convergence.

Furthermore if \(c(\cdot ) \in E_{\gamma }(\Omega )\) and \(\gamma \prec \prec \psi \) then \(N_{f}\) is strongly continuous from \({\mathcal {P}}\left( E_{M}(\Omega ), \frac{1}{k_{2}}\right) ^{p}\) to \(\left( E_{\gamma }(\Omega )\right) ^{q}\)

Lemma 3.9

Let \(\Omega \) be a bounded open subset of \( R^{N} \) with the segment property. If \( u\in (W^{1}_{0}L_{\varphi }(\Omega ))^{N} \) then \(\displaystyle { \int _{\Omega } \text{ div } u \ dx = 0} \).

Proof of lemma 3.9

The proof of this lemma is based on [30], Lemma 3.2 ]

4 Main result

We consider the following boundary value problem

$$\begin{aligned} ({\mathcal {P}})\left\{ \begin{array}{c} {A(u)-{\text {div}}\Phi (u)+g(., u, \nabla u)=f \in W^{-1} E_{\psi }(\Omega ) , \quad \text{ in } \Omega } \\ \qquad \qquad \qquad {u \equiv 0, \quad \text{ on } \partial \Omega } \end{array}\right. \end{aligned}$$

Let us define

$$\begin{aligned} {\mathcal {T}}_{0}^{1, \varphi }(\Omega )=\left\{ u \text{ measurable } \quad \text{ such } \text{ that } \quad T_{k}(u) \in W_{0}^{1} L_{\varphi }(\Omega ), \forall k>0\right\} . \end{aligned}$$

As in [21] we define the following notion of renormalized solution, which gives a meaning to a possible solution of \(({\mathcal {P}})\)

Definition 4.1

Assume that (1.2)–(1.4), (1.6) hold true. A function u is a renormalized solution of the problem \(({\mathcal {P}})\) if

$$\begin{aligned} \left\{ \begin{aligned} u \in {\mathcal {T}}_{0}^{1, \varphi }(\Omega ), g(., u, \nabla u) \in L^{1}(\Omega ), g(., u, \nabla u) u \in L^{1}(\Omega )\qquad \qquad \qquad \qquad \qquad \qquad \\ \int _{\Omega } a(x, u, \nabla u) h(u) \nabla v d x+\int _{\Omega } a(x, u, \nabla u) h^{\prime }(u) \nabla u v d x\\ + \int _{\Omega } \Phi (u) h(u)\nabla v d x+\int _{\Omega } \Phi ( u) h^{\prime }(u) \nabla u v d x \\ + \int _{\Omega } g(x, u, \nabla u) h(u) v d x =\int _{\Omega } f h(u) v d x \qquad \qquad \qquad \qquad \qquad \qquad \end{aligned}\right. \end{aligned}$$
(4.1)

for all \(h \in W^{1, \infty }({\mathbb {R}})\) such that \(h^{\prime }\) has a compact support in \({\mathbb {R}}\), and for all \(v \in W_{0}^{1} L_{\varphi }(\Omega ) \cap L^{\infty }(\Omega )\).

The weaker problem (4.1) is obtained by using the test function h(u)v where \(h \in \) \(W^{1, \infty }({\mathbb {R}}) .\) and \(v \in W_{0}^{1} L_{\varphi }(\Omega ) \cap L^{\infty }(\Omega )\) in \(({\mathcal {P}})\).

Remark 1

Let us note that in (4.1) every term is meaningful in the distributional sense.

Theorem 4.1

Under assumptions (1.2)–(1.4),(1.6) there exists at least a renormalized solution u in the sense of definition 4.1 of problem \(({\mathcal {P}})\).

Let us introduce the truncate operator. For a given constant \(k> 0\), we define the function \(T_{k}: {\mathbb {R}}\rightarrow {\mathbb {R}}\) as

$$\begin{aligned} T_{k}(s)=\left\{ \begin{array}{rcl} s \text{ if } \left| s\right| \le k,\\ k\frac{s}{\left| s\right| } \text{ if } \left| s\right| > k. \end{array}\right. \end{aligned}$$

4.1 Proof of Theorem 4.1

4.1.1 Approximate problem and a priori estimate

We use an idea contained in [37] (Theorem 1.1), based on the approximation of the original problem and a priori estimate. For \(n \in {\mathbb {N}}\), let \(\left( f_{n}\right) _{n}\) be a sequence in \(W^{-1} E_{\psi }(\Omega ) \cap L^{1}(\Omega )\) such that \(f_{n} \longrightarrow f\) in \(L^{1}(\Omega )\) with \(\left\| f_{n}\right\| _{1} \le \Vert f\Vert _{1}, \phi _{n}(s)=\phi \left( T_{n}(s)\right) \) and \(g_{n}(x, s, \xi )=T_{n}(g(x, s, \xi ))\). The following approximate problem

$$\begin{aligned} (P_{n})\left\{ \begin{array}{c} {-{\text {div}}\left( a\left( \cdot , u_{n}, \nabla u_{n}\right) \right) +g_{n}\left( \cdot , u_{n}, \nabla u_{n}\right) =f_{n} +{\text {div}}(\Phi _{n}(u_{n})) \quad \text{ in } D^{\prime }(\Omega )}\\ \\ \qquad \qquad \qquad \qquad \qquad \qquad u_{n} =0 \qquad \qquad \qquad \qquad \qquad \text{ on } \partial \Omega , \end{array}\right. \end{aligned}$$

has a solution \(u_{n}\) in \( W^1_0L_\varphi (\Omega )\) .

Now Choosing \(u_{n}\) as a function test in problem \( (P_{n}),\) we have

$$\begin{aligned} \int _{\Omega } a\left( x, u_{n}, \nabla u_{n}\right) \cdot \nabla u_{n} d x+\int _{\Omega } \Phi _{n}\left( u_{n}\right) \cdot \nabla u_{n} d x+\int _{\Omega } g_{n}\left( x, u_{n}, \nabla u_{n}\right) u_{n} d x=\left\langle f, u_{n}\right\rangle \end{aligned}$$
(4.2)

By posing

$$\begin{aligned} {\widetilde{\Phi }}_{n}(t)=\int _{0}^{t} \Phi _{n}(\tau ) d \tau \end{aligned}$$

we obtain

$$\begin{aligned} {\bar{\Phi }}_{n}(0)=0 . \end{aligned}$$

As each component of \({\bar{\Phi }}_{n}\) is uniformly Lipschitizian, and according to [32], Lemma 2], it follows that the function \({\bar{\Phi }}_{n}\left( u_{n}\right) \) belongs to \(\left( W_{0}^{1} L_{\varphi }(\Omega )\right) ^{N} .\)

therefore by using Lemma 3.9

$$\begin{aligned} \int _{\Omega } \Phi _{n}\left( u_{n}\right) \cdot \nabla u_{n} d x=\int _{\Omega } {\text {div}}\left( {\widetilde{\Phi }}_{n}\left( u_{n}\right) \right) d x=0 \end{aligned}$$

According to (1.7) and using Young’s inequality, we have

$$\begin{aligned} \left| \int _{\Omega } a\left( x, u_{n}, \nabla u_{n}\right) \cdot \nabla u_{n} d x \right| \le C_{1}+\frac{\alpha }{2} \int _{\Omega } \varphi \left( x,\left| \nabla T_{k}\left( u_{n}\right) \right| \right) d x. \end{aligned}$$
(4.3)

which together with (1.5) gives

$$\begin{aligned} \displaystyle { \int _{\Omega } \varphi \left( x,\left| \nabla T_{k}\left( u_{n}\right) \right| \right) d x\le C_{2}} \end{aligned}$$
(4.4)

Poincare inequality (see Lemma3.3) implies that

$$\begin{aligned} \int _{\Omega } \varphi \left( x, \frac{\left| T_{k}\left( u_{n}\right) \right| }{c}\right) d x \le \int _{\Omega } \varphi \left( x,\left| \nabla T_{k}\left( u_{n}\right) \right| \right) d x \le c_{2} k \end{aligned}$$
(4.5)

On the other hand we have

$$\begin{aligned} {\qquad \int _{\Omega } g_{n}\left( x, u_{n}, \nabla u_{n}\right) u_{n} d x \le C_{3}} \end{aligned}$$
(4.6)

so it follows that \(\left( T_{k}\left( u_{n}\right) \right) _{n}\) and \(\left( \nabla T_{k}\left( u_{n}\right) \right) _{n}\) are bounded in \(L_{\varphi }(\Omega ),\) Thus

$$\begin{aligned}\left( T_{k}\left( u_{n}\right) \right) _{n} is \text{ bounded } \text{ in } W_{0}^{1} L_{\varphi }(\Omega ),\end{aligned}$$

there exists some \(v_{k} \in W_{0}^{1} L_{\varphi }(\Omega )\) such that

$$\begin{aligned} \left\{ \begin{aligned} T_{k}\left( u_{n}\right) \rightharpoonup v_{k} \quad \text{ weakly } \text{ in } W_{0}^{1} L_{\varphi }(\Omega ) \text{ for } \sigma \left( \Pi L_{\varphi }, \Pi E_{\psi }\right) \\ T_{k}\left( u_{n}\right) \longrightarrow v_{k} \qquad \text{ strongly } \text{ in } E_{\psi }(\Omega ). \qquad \qquad \quad \qquad \end{aligned}\right. \end{aligned}$$
(4.7)

Now one suppose that exists a function \(\varphi \) satisfies \(\lim _{t \rightarrow \infty } \frac{\varphi (t)}{t}=\infty \) and \(\varphi (t) \le {\text {ess}} \inf _{x \in \Omega } \varphi (x, t)\) Let \(k>0\) large enough, by using (4.5) we have

$$\begin{aligned} \begin{aligned} \varphi (k) {\text {meas}}\left\{ \left| u_{n}\right|>k\right\}&=\int _{\left\{ \left| u_{n}\right|>k\right\} } \varphi \left( \left| T_{k}\left( u_{n}\right) \right| \right) d x \\&\le \int _{\left\{ \left| u_{n}\right| >k\right\} } \varphi \left( x,\left| T_{k}\left( u_{n}\right) \right| \right) d x \le \int _{\Omega } \varphi \left( x,\left| T_{k}\left( u_{n}\right) \right| \right) d x \\&\le c_{3} k \end{aligned} \end{aligned}$$

Hence

$$\begin{aligned} \hbox {meas}\left\{ \left| u_{n}\right| >k\right\} \le \frac{c_{3} k}{\varphi (k)} \longrightarrow 0 \text{ as } k \longrightarrow \infty \end{aligned}$$

For every \(\lambda >0,\) we have

$$\begin{aligned} \begin{aligned} \left. \hbox {meas}\left\{ \left| u_{n}-u_{m}\right|>\lambda \right\} \le \lambda \right\}&\le \hbox {meas}\left\{ \left| u_{n}\right|>k\right\} +\hbox {meas}\left\{ \left| u_{m}\right|>k\right\} \\&\quad +\hbox {meas}\left\{ \left| T_{k}\left( u_{n}\right) -T_{k}\left( u_{m} \right) \right| >\lambda \right\} \end{aligned} \end{aligned}$$
(4.8)

then, by using (4.5) one suppose that \(\left( T_{k}\left( u_{n}\right) \right) _{n}\) is a Cauchy sequence in measure in \(\Omega \), Let \(\varepsilon >0,\) then by (4.8) there exists some \(k=k(\varepsilon )>0\) such that

$$\begin{aligned} \hbox {meas}\left\{ \left| u_{n}-u_{m}\right| >\lambda \right\} <\varepsilon , \quad \text{ for } \text{ all } n, m \ge h_{0}(k(\varepsilon ), \lambda ) \end{aligned}$$

which means that \(\left( u_{n}\right) _{n}\) is a Cauchy sequence in measure in \(\Omega ,\) thus converge almost every where to u.

Consequently

$$\begin{aligned} \left\{ \begin{aligned} {u_{n} \rightharpoonup u}&\text{ weakly } \text{ in } W_{0}^{1} L_{\varphi }(\Omega ) \text{ for } \sigma \left( \Pi L_{\varphi }, \Pi E_{\psi }\right) \\ {u_{n} \longrightarrow u}&\text{ strongly } \text{ in } E_{\psi }(\Omega ). \end{aligned}\right. \end{aligned}$$
(4.9)

4.1.2.

In this step we shall show the boundedness of \(\left( a\left( \cdot , T_{k}\left( u_{n}\right) , \nabla T_{k}\left( u_{n}\right) \right) \right) _{n}\) in \(\left( L_{\psi }(\Omega )\right) ^{N}\)

Let \(\vartheta \in E_{\varphi }(\Omega )^{N}\) such that \( \Vert \vartheta \Vert _{\varphi , \Omega } \le 1, \) the hypothesis (1.4) gives We have

$$\begin{aligned} \int _{\Omega }\left[ a\left( x, T_{k}\left( u_{n}\right) , \nabla T_{k}\left( u_{n}\right) \right) -a\left( x, T_{k}\left( u_{n}\right) , \frac{\vartheta }{k_{3}}\right) \right] \left[ \nabla T_{k}\left( u_{n}\right) -\frac{\vartheta }{k_{3}}\right] d x > 0 \end{aligned}$$

This implies that

$$\begin{aligned}&\int _{\Omega } \frac{1}{k_{3}} a\left( x, T_{k}\left( u_{n}\right) , \nabla T_{k}\left( u_{n}\right) \right) \vartheta d x \\&\quad \le \int _{\Omega } a\left( x, T_{k}\left( u_{n}\right) , \nabla T_{k}\left( u_{n}\right) \right) \nabla T_{k}\left( u_{n}\right) d x \\&\qquad -\int _{\Omega } a\left( x, T_{k}\left( u_{n}\right) , \frac{\vartheta }{k_{3}}\right) \left( \nabla T_{k}\left( u_{n}\right) -\frac{\vartheta }{k_{3}}\right) d x \\&\quad \le c_{2}k-\int _{\Omega } a\left( x, T_{k}\left( u_{n}\right) , \frac{\vartheta }{k_{3}}\right) \nabla T_{k}\left( u_{n}\right) d x \\&\qquad +\frac{1}{k_{3}} \int _{\Omega } a\left( x, T_{k}\left( u_{n}\right) , \frac{\vartheta }{k_{3}}\right) \vartheta d x \end{aligned}$$

By using Young’s inequality in the last two terms of the last side and (4.5) we get

$$\begin{aligned}&\int _{\Omega } a\left( x, T_{k}\left( u_{n}\right) , \nabla T_{k}\left( u_{n}\right) \right) \vartheta d x \le c_{2}k k_{3} \\&\qquad +3 k_{1}\left( 1+k_{3}\right) \int _{\Omega } \psi \left( x, \frac{\left| a\left( x, T_{k}\left( u_{n}\right) , \frac{\vartheta }{k_{3}}\right) \right| }{3 k_{1}}\right) d x\\&\qquad +3 k_{1} k_{3} \int _{\Omega } \varphi \left( x,\left| \nabla T_{k}\left( u_{n}\right) \right| \right) d x+3 k_{1} \int _{\Omega } \varphi (x,|\vartheta |) d x \\&\quad \le c_{2}k k_{3}+3 k_{1} k_{3} c_{2}k +3 k_{1} \\&\qquad +3 k_{1}\left( 1+k_{3}\right) \int _{\Omega } \psi \left( x, \frac{\left| a\left( x, T_{k}\left( u_{n}\right) , \frac{\vartheta }{k_{3}}\right) \right| }{3 k_{1}}\right) d x \end{aligned}$$

Now, by using (1.3) and the convexity of \(\psi \) we get

$$\begin{aligned} \psi \left( x, \frac{\left| a\left( x, T_{k}\left( u_{n}\right) , \frac{\vartheta }{k_{3}}\right) \right| }{3 k_{1}}\right) \le \frac{1}{3}\left( \psi (x, d(x))+\gamma \left( x, k_{2}\left| T_{k}\left( u_{n}\right) \right| \right) +\varphi (x,|\vartheta |)\right) . \end{aligned}$$

Thanks to “Remark 2.1” there exists \(h \in L^{1}(\Omega )\) such that

$$\begin{aligned} \gamma \left( x, k_{2}\left| T_{k}\left( u_{n}\right) \right| \right) \le \gamma \left( x, k_{2} k\right) \le \varphi (x, 1)+h(x) ; \end{aligned}$$

then by integrating over \(\Omega \) we deduce that

$$\begin{aligned} \begin{array}{l}\displaystyle { \int _{\Omega } \psi \left( x, \frac{\left| a\left( x, T_{k}\left( u_{n}\right) , \frac{v}{k_{3}}\right) \right| }{3 k_{1}}\right) d x \le \frac{1}{3}\left( \int _{\Omega } \psi (x, d(x)) d x+\int _{\Omega } h(x) d x \right. } \\ \displaystyle { \left. \quad +\int _{\Omega } \varphi (x, 1) d x+\int _{\Omega } \varphi (x,|\vartheta |) d x\right) \le c_{k} }, \end{array} \end{aligned}$$

where \(c_{k} \) is a constant depending on k. So,

$$\begin{aligned} \int _{\Omega } a\left( x, T_{k}\left( u_{n}\right) , \nabla T_{k}\left( u_{n}\right) \right) \vartheta d x \le c_{k}, \quad \forall \vartheta \in \left( E_{\varphi }(\Omega )\right) ^{N} \quad \text{ with } \Vert \vartheta \Vert _{\varphi , \Omega }=1 \end{aligned}$$

and thus \(\left\| a\left( x, T_{k}\left( u_{n}\right) , \nabla T_{k}\left( u_{n}\right) \right) \right\| _{\psi , \Omega } \le c_{k},\) which implies that,

$$\begin{aligned} \left( a\left( x, T_{k}\left( u_{n}\right) , \nabla T_{k}\left( u_{n}\right) \right) \right) _{n} \text { is bounded in } L_{\psi }(\Omega )^{N}. \end{aligned}$$
(4.10)

4.1.3.

Let us show that :

$$\begin{aligned} \displaystyle {\qquad \lim _{m \rightarrow \infty } \lim _{n \rightarrow \infty } \int _{\left( m \le \left| u_{n}\right| \le m+1 |\right. } a\left( x, u_{n}, \nabla u_{n}\right) \cdot \nabla u_{n} d x=0} \end{aligned}$$

Defining

$$\begin{aligned} \theta _{m}(r)=T_{m+1}(r)-T_{m}(r)\qquad \text{ For } \text{ any } m \ge 1, \end{aligned}$$

in view of [32], Lemma2] one get \(\theta _{m}\left( u_{n}\right) \in W_{0}^{1} L_{\varphi }(\Omega ). \)

Now let us taking \(\theta _{m}\left( u_{n}\right) \) as a test function in \( (P_{n})\) we have

$$\begin{aligned}&\int _{\Omega } a\left( x, u_{n}, \nabla u_{n}\right) \nabla \theta _{m}\left( u_{n}\right) d x+\int _{\Omega } \Phi _{n}\left( u_{n}\right) \nabla \theta _{m}\left( u_{n}\right) d x\\&\quad + \int _{\Omega }g_{n}\left( x, u_{n}, \nabla u_{n}\right) \theta _{m}\left( u_{n}\right) d x =\int _{\Omega } f_{n} \theta _{m}\left( u_{n}\right) d x \end{aligned}$$

Consider,

$$\begin{aligned} \begin{array}{l} \phi (t)=\Phi _{n}(t) \chi _{\{s \in {\mathbb {R}},m\le |s|\le m+1 \}}(t) \\ {\tilde{\phi }}(t)=\displaystyle {\int _{0}^{t} \phi (\tau ) d \tau } \end{array} \end{aligned}$$

hence \({\tilde{\phi }}\left( u_{n}\right) \in \left( W_{0}^{1} L_{\varphi }(\Omega )\right) ^{N}\) (by Lemma3.2). We obtain, by Lemma 3.9,

$$\begin{aligned} \begin{aligned} \int _{\Omega } \Phi _{n}\left( u_{n}\right) \nabla \theta _{m}\left( u_{n}\right) d x&=\int _{\Omega } \Phi _{n}\left( u_{n}\right) \chi _{\{s \in {\mathbb {R}},m\le |s|\le m+1 \}}\left( u_{n}\right) \nabla u_{n} d x \\&=\int _{\Omega } \phi \left( u_{n}\right) \nabla u_{n} d x=\int _{\Omega } {\text {div}}\left( {\tilde{\phi }}\left( u_{n}\right) \right) d x=0 \end{aligned} \end{aligned}$$

Using the sign condition (1.7) we have \(g_{n}\left( x, u_{n}, \nabla u_{n}\right) \theta _{m}\left( u_{n}\right) \ge 0\) a.e. in \(\Omega ,\) and knowing that \(\nabla \theta _{m}\left( u_{n}\right) =\nabla u_{n} \chi _{\left\{ m \le \left| u_{n}\right| \le m+1\right\} } \text{ a.e. } \text{ in } \Omega , \) we get

$$\begin{aligned} \int _{\{m\le |u_{n}|\le m+1 \}} a\left( x, u_{n}, \nabla u_{n}\right) \nabla u_{n} d x \le \left\langle f, \theta _{m}\left( u_{n}\right) \right\rangle . \end{aligned}$$

It is not difficult to see that

$$\begin{aligned}\left\| \nabla \theta _{m}\left( u_{n}\right) \right\| _{\varphi ,\Omega } \le \left\| \nabla u_{n}\right\| _{\varphi ,\Omega } . \end{aligned}$$

then in view of (4.4) and (4.9) it follows that

$$\begin{aligned} \theta _{m}\left( u_{n}\right) \rightharpoonup \theta _{m}(u) \quad \text{ weakly } \text{ in } W_{0}^{1} L_{\varphi }(\Omega ) \quad \text{ for } \sigma \left( \Pi L_{\varphi }(\Omega ), \Pi E_{\varphi }(\Omega )\right) \end{aligned}$$

Therefore, we get

$$\begin{aligned} \lim _{n \rightarrow \infty } \int _{\left\{ m \le \left| u_{u}\right| \le m+1\right\} } a\left( x, u_{n}, \nabla u_{n}\right) \cdot \nabla u_{n} d x \le \left\langle f, \theta _{m}(u)\right\rangle \end{aligned}$$

as \(\theta _{m}(u) \rightharpoonup 0\) weakly in \(W_{0}^{1} L_{\varphi }(\Omega ,)\) for \(\sigma \left( \Pi L_{\varphi }(\Omega ), \Pi E_{\varphi }(\Omega )\right) \) one obtain

$$\begin{aligned} \lim _{m \rightarrow \infty } \lim _{n \rightarrow \infty } \int _{\left( m \le || n_{n}|\le m+1|\right. } a\left( x, u_{n}, \nabla u_{n}\right) \cdot \nabla u_{n} d x \le \lim _{m \rightarrow \infty }\left\langle f, \theta _{m}(u)\right\rangle = 0 \end{aligned}$$

By (1.5), we get

$$\begin{aligned} \displaystyle {\qquad \lim _{m \rightarrow \infty } \lim _{n \rightarrow \infty } \int _{\left( m \le \left| u_{k}\right| \le m+1 |\right. } a\left( x, u_{n}, \nabla u_{n}\right) \cdot \nabla u_{n} d x=0} \end{aligned}$$
(4.11)

4.1.4.

In this subsubsection we pose \(\phi (s)=s e^{\lambda s^{2}}\) where \(\lambda =\left( \frac{b(k)}{2 \alpha }\right) ^{2}.\) it is easy to get,

$$\begin{aligned} \text{ for } \text{ all } s \in {\mathbb {R}},\qquad \phi ^{\prime }(s)-\frac{b(k)}{\alpha }|\phi (s)| \ge \frac{1}{2} \end{aligned}$$
(4.12)

For \(m \ge k,\) definning

$$\begin{aligned} \psi _{m}(s)=\left\{ \begin{array}{ll} {1} &{} { \text{ if } |s| \le m} \\ {m+1-|s|} &{} { \text{ if } m \le |s| \le m+1} \\ {0} &{} { \text{ if } |s| \ge m+1} \end{array}\right. \end{aligned}$$

Let \(\left\{ v_{j}\right\} _{j} \subset D(\Omega )\) be a sequence such that \(v_{j} \rightarrow u\) in \(W_{0}^{1} L_{\varphi }(\Omega )\) for the modular convergence and a e. in \(\Omega \). And let us define the functions

$$\begin{aligned} \theta _{n}^{j}=T_{k}\left( u_{n}\right) -T_{k}\left( v_{j}\right) , \theta ^{j}=T_{k}(u)-T_{k}\left( v_{j}\right) \text{ and } z_{n, m}^{j}=\phi \left( \theta _{n}^{j}\right) \psi _{m}\left( u_{n}\right) . \end{aligned}$$

Using \(z_{n, m}^{j} \in W_{0}^{1} L_{\varphi }(\Omega )\) as a test function in \( (P_{n}) \) we get

$$\begin{aligned}&\int _{\Omega }a\left( x, u_{n}, \nabla u_{n}\right) \cdot \nabla z_{n, m}^{j} d x\nonumber \\&\quad +\int _{\left\{ m \le \left| u_{n}\right| \le m+1\right\} } \Phi _{n}\left( u_{n}\right) \cdot \nabla u_{n} \psi _{m}^{\prime }\left( u_{n}\right) \phi \left( T_{k}\left( u_{n}\right) -T_{k}\left( v_{j}\right) \right) d x\nonumber \\&\quad +\int _{\Omega } \Phi _{n}\left( u_{n}\right) \cdot \nabla \phi \left( T_{k}\left( u_{n}\right) -T_{k}\left( v_{j}\right) \right) \psi _{m}\left( u_{n}\right) d x\nonumber \\&\quad +\int _{\Omega } g_{n}\left( x, u_{n}, \nabla u_{n}\right) z_{n, m}^{j} d x=\int _{\Omega } f z_{n, m}^{j} d x \end{aligned}$$
(4.13)

From now on, we denote by \(\epsilon _{i}(n, j), i=0,1,2, \ldots ,\) various sequences of real numbers which tend to zero as n and \(j \rightarrow \infty ,\) i.e.,

$$\begin{aligned} \lim _{j \rightarrow +\infty } \lim _{n \rightarrow +\infty } \epsilon _{i}(n, j)=0 \end{aligned}$$

by using (4.7) one has \(z_{n, m}^{j} \rightarrow \phi \left( \theta ^{j}\right) \psi _{m}(u)\) weakly in \(L^{\infty }(\Omega )\) for \(\sigma ^{*}\left( L^{\infty }, L^{1}\right) \) as \(n \rightarrow \infty \) which give

$$\begin{aligned} \lim _{n \rightarrow \infty } \int _{\Omega } f z_{n, m}^{j} d x=\int _{\Omega } f \phi \left( \theta ^{j}\right) \psi _{m}(u) d x \end{aligned}$$

and \(\phi \left( \theta ^{j}\right) \rightarrow 0\) weakly in \(L^{\infty }(\Omega )\) for \(\sigma \left( L^{\infty }, L^{1}\right) \) as \(j \rightarrow \infty ,\) we have

$$\begin{aligned} \lim _{j \rightarrow \infty } \int _{\Omega } f \phi \left( \theta ^{j}\right) \psi _{m}(u) d x=0 \end{aligned}$$

Therefore, by denoting

$$\begin{aligned} \int _{\Omega } f z_{n, m}^{j} d x=\epsilon _{0}(n, j), \end{aligned}$$

the divergence lemma implies that

$$\begin{aligned} \int _{\left\{ m \le \left| u_{n}\right| \le m+1\right\} } \Phi _{n}\left( u_{n}\right) \cdot \nabla u_{n} \psi _{m}^{\prime }\left( u_{n}\right) \phi \left( T_{k}\left( u_{n}\right) -T_{k}\left( v_{j}\right) \right) d x=0. \end{aligned}$$

The third term in the left-hand side of (4.13) can be written as follows

$$\begin{aligned} \begin{aligned}&\int _{\Omega } \Phi _{n}\left( u_{n}\right) \cdot \nabla \phi \left( T_{k}\left( u_{n}\right) -T_{k}\left( v_{j}\right) \right) \psi _{m}\left( u_{n}\right) d x \\&\quad =\int _{\Omega } \Phi _{n}\left( u_{n}\right) \cdot \nabla T_{k}\left( u_{n}\right) \phi ^{\prime }\left( \theta _{n}^{j}\right) \psi _{m}\left( u_{n}\right) d x\\&\qquad -\int _{\Omega } \Phi _{n}\left( u_{n}\right) \cdot \nabla T_{k}\left( v_{j}\right) \phi ^{\prime }\left( \theta _{n}^{j}\right) \psi _{m}\left( u_{n}\right) d x \end{aligned} \end{aligned}$$

Applying the divergence lemma we have,

$$\begin{aligned} \int _{\Omega } \Phi _{n}\left( u_{n}\right) \cdot \nabla T_{k}\left( u_{n}\right) \phi ^{\prime }\left( \theta _{n}^{j}\right) \psi _{m}\left( u_{n}\right) d x=0. \end{aligned}$$

By (4.7) one obtain

$$\begin{aligned} \Phi _{n}\left( u_{n}\right) \phi ^{\prime }\left( \theta _{n}^{j}\right) \psi _{m}\left( u_{n}\right) \rightarrow \Phi (u) \phi ^{\prime }\left( \theta ^{j}\right) \psi _{m}(u) \text{ a.e. } \text{ in } \Omega \quad as\quad n \rightarrow +\infty \end{aligned}$$

now, we can verify that

$$\begin{aligned} \left\| \Phi _{n}\left( u_{n}\right) \phi ^{\prime }\left( \theta _{n}^{j}\right) \psi _{m}\left( u_{n}\right) \right\| _{\varphi ,\Omega } \le \psi \left( x,c_{m} \phi ^{\prime }(2 k)\right) |\Omega |+1 \end{aligned}$$

with \(c_{m}=\max _{|t| \le m+1} \Phi (t)\).

Thanks to [33], Theorem 14.6], we have

$$\begin{aligned} \lim _{n \rightarrow +\infty } \int _{\Omega } \Phi _{n}\left( u_{n}\right) \cdot \nabla T_{k}\left( v_{j}\right) \phi ^{\prime }\left( \theta _{n}^{j}\right) \psi _{m}\left( u_{n}\right) d x=\int _{\Omega } \Phi (u) \cdot \nabla T_{k}\left( v_{j}\right) \phi ^{\prime }\left( \theta ^{j}\right) \psi _{m}(u) d x \end{aligned}$$

Using the modular convergence of the sequence \(\left\{ v_{j}\right\} _{j},\) it follows that

$$\begin{aligned} \lim _{j \rightarrow +\infty } \lim _{n \rightarrow +\infty } \int _{\Omega } \Phi _{n}\left( u_{n}\right) \cdot \nabla T_{k}\left( v_{j}\right) \phi ^{\prime }\left( \theta _{n}^{j}\right) \psi _{m}\left( u_{n}\right) d x=\int _{\Omega } \Phi (u) \cdot \nabla T_{k}(u) \psi _{m}(u) d x \end{aligned}$$

Then, thanks to Lemma 3.9 we obtain

$$\begin{aligned} \int _{\Omega } \Phi (u) \cdot \nabla T_{k}(u) \psi _{m}(u) d x=0 \end{aligned}$$

Therefore,we denote

$$\begin{aligned} \int _{\Omega } \Phi _{n}\left( u_{n}\right) \cdot \nabla \phi \left( T_{k}\left( u_{n}\right) -T_{k}\left( v_{j}\right) \right) \psi _{m}\left( u_{n}\right) d x=\epsilon _{1}(n, j). \end{aligned}$$

since \(g_{n}\left( x, u_{n}, \nabla u_{n}\right) z_{n m}^{j} \ge 0\) on the set \(\left\{ \left| u_{n}\right| >k\right\} \) and \(\psi _{m}\left( u_{n}\right) =1\) on the set \(\left\{ \left| u_{n}\right| \le k\right\} ,\) by according to 4.13 we get

$$\begin{aligned} \displaystyle { \int _{\Omega } a\left( x, u_{n}, \nabla u_{n}\right) \cdot \nabla z_{n m}^{j} d x+\int _{\left\{ \left| u_{n}\right| \le k |\right. } g_{n}\left( x, u_{n}, \nabla u_{n}\right) \phi \left( \theta _{n}^{j}\right) d x \le \epsilon _{2}(n, j) } \end{aligned}$$
(4.14)

For the first term of the left-hand side of (4.14) we can write

$$\begin{aligned}&\int _{\Omega } a\left( x, u_{n}, \nabla u_{n}\right) \cdot \nabla z_{n, m}^{j} d x =\int _{\Omega } a\left( x, u_{n}, \nabla u_{n}\right) \cdot \left( \nabla T_{k}\left( u_{n}\right) \right. \\&\qquad \left. -\nabla T_{k}\left( v_{j}\right) \right) \phi ^{\prime }\left( \theta _{n}^{j}\right) \psi _{m}\left( u_{n}\right) d x\\&\qquad +\int _{\Omega } a\left( x, u_{n}, \nabla u_{n}\right) \cdot \nabla u_{n} \phi \left( \theta _{n}^{j}\right) \psi _{m}^{\prime }\left( u_{n}\right) d x\\&\quad =\int _{\Omega } a\left( x, T_{k}\left( u_{n}\right) , \nabla T_{k}\left( u_{n}\right) \right) \cdot \left( \nabla T_{k}\left( u_{n}\right) -\nabla T_{k}\left( v_{j}\right) \right) \phi ^{\prime }\left( \theta _{n}^{j}\right) d x\\&\qquad -\int _{|| u_{n}|>k|} a\left( x, u_{n}, \nabla u_{n}\right) \cdot \nabla T_{k}\left( v_{j}\right) \phi ^{\prime }\left( \theta _{n}^{j}\right) \psi _{m}\left( u_{n}\right) d x\\&\qquad +\int _{\Omega } a\left( x, u_{n}, \nabla u_{n}\right) \cdot \nabla u_{n} \phi \left( \theta _{n}^{j}\right) \psi _{m}^{\prime }\left( u_{n}\right) d x \end{aligned}$$

therefore

$$\begin{aligned}&\int _{\Omega } a\left( x, u_{n}, \nabla u_{n}\right) \cdot \nabla z_{n m}^{j} d x\nonumber \\&\quad =\int _{\Omega } \left( a\left( x, T_{k}\left( u_{n}\right) , \nabla T_{k}\left( u_{n}\right) \right) -a\left( x, T_{k}\left( u_{n}\right) , \nabla T_{k}\left( v_{j}\right) \chi _{j}^{s}\right) \right) \nonumber \\&\qquad \left( \nabla T_{k}\left( u_{n}\right) -\nabla T_{k}\left( v_{j}\right) \chi _{j}^{s}\right) \phi ^{\prime }\left( \theta _{n}^{j}\right) d x\nonumber \\&\qquad +\int _{\Omega } a\left( x, T_{k}\left( u_{n}\right) , \nabla T_{k}\left( v_{j}\right) \chi _{j}^{s}\right) \cdot \left( \nabla T_{k}\left( u_{n}\right) -\nabla T_{k}\left( v_{j}\right) \chi _{j}^{\prime }\right) \phi ^{\prime }\left( \theta _{n}^{j}\right) d x\nonumber \\&\qquad -\int _{\Omega \backslash \Omega _{j}^{*}} a\left( x, T_{k}\left( u_{n}\right) , \nabla T_{k}\left( u_{n}\right) \right) \cdot \nabla T_{k}\left( v_{j}\right) \phi ^{\prime }\left( \theta _{n}^{j}\right) d x\nonumber \\&\qquad -\int _{\left[ \left| u_{u}\right| >k\right] } a\left( x, u_{n}, \nabla u_{n}\right) \cdot \nabla T_{k}\left( v_{j}\right) \phi ^{\prime }\left( \theta _{n}^{j}\right) \psi _{m}\left( u_{n}\right) d x\nonumber \\&\qquad +\int _{\Omega } a\left( x, u_{n}, \nabla u_{n}\right) \cdot \nabla u_{n} \phi \left( \theta _{n}^{j}\right) \psi _{m}^{\prime }\left( u_{n}\right) d x \end{aligned}$$
(4.15)

let us define \(x_{j}^{s}, s>0,\) and the characteristic function of the subset \(\Omega _{j}^{s}=\left\{ x \in \Omega :\left| \nabla T_{k}\left( v_{j}\right) \right| \le s\right\} \).

By fixing m and s,  we will pass to the limit in n and in j in the second, third, fourth and fifth term on the right hand side of (4.15) .

For the second term, we have

$$\begin{aligned}&\int _{\Omega } a\left( x, T_{k}\left( u_{n}\right) , \nabla T_{k}\left( v_{j}\right) \chi _{j}^{s}\right) \cdot \left( \nabla T_{k}\left( u_{n}\right) -\nabla T_{k}\left( v_{j}\right) \chi _{j}^{s}\right) \phi ^{\prime }\left( \theta _{n}^{j}\right) d x\\&\quad \rightarrow \int _{\Omega } (a\left( x, T_{k}(u), \nabla T_{k}\left( v_{j}\right) \chi _{j}^{s}\right) \cdot \left( \nabla T_{k}(u)-\nabla T_{k}\left( v_{j}\right) \chi _{j}^{s}\right) \phi ^{\prime }\left( \theta ^{j}\right) ) d x \quad \hbox { as }n \rightarrow +\infty \end{aligned}$$

thinks to 3.8, one has

$$\begin{aligned}&a\left( x, T_{k}\left( u_{n}\right) , \nabla T_{k}\left( v_{j}\right) x_{j}^{s}\right) \phi ^{\prime }\left( \theta _{n}^{j}\right) \rightarrow \\&a\left( x, T_{k}(u), \nabla T_{k}\left( v_{j}\right) \chi _{j}^{s}\right) \phi ^{\prime }\left( \theta ^{j}\right) \text{ strongly } \text{ in } \left( E_{\varphi }(\Omega )\right) ^{N} \text{ as } \text{ n } \rightarrow \infty \end{aligned}$$

and by (4.4)

$$\begin{aligned} \nabla T_{k}\left( u_{n}\right) \rightharpoonup \nabla T_{k}(u) \quad \text{ weakly } \text{ in } \left( L_{\varphi }(\Omega )\right) ^{N} \end{aligned}$$

Let us define \(\chi ^{s}\) the characteristic function of the subset

$$\begin{aligned} \Omega ^{s}=\left\{ x \in \Omega :\left| \nabla T_{k}(u)\right| \le s\right\} \end{aligned}$$

As \(\nabla T_{k}\left( v_{j}\right) \chi _{j}^{s} \rightarrow \nabla T_{k}(u) \chi ^{s}\) strongly in \(\left( E_{\varphi }(\Omega )\right) ^{N}\) as \(j \rightarrow \infty ,\) we get

$$\begin{aligned} \int _{\Omega } a\left( x, T_{k}(u), \nabla T_{k}\left( v_{j}\right) \chi _{j}^{s}\right) \cdot \left( \nabla T_{k}(u)-\nabla T_{k}\left( v_{j}\right) \chi _{j}^{s}\right) \phi ^{\prime }\left( \theta ^{j}\right) d x \rightarrow 0 \quad \text{ as } j \rightarrow \infty \end{aligned}$$

thus,

$$\begin{aligned} \displaystyle {\int _{\Omega } a\left( x, T_{k}\left( u_{n}\right) , \nabla T_{k}\left( v_{j}\right) \chi _{j}^{s}\right) \cdot \left( \nabla T_{k}\left( u_{n}\right) -\nabla T_{k}\left( v_{j}\right) \chi _{j}^{s}\right) \phi ^{\prime }\left( \theta _{n}^{j}\right) ) d x=\epsilon _{3}(n, j)} \end{aligned}$$
(4.16)

For third term estimation of (4.15) . It’s it is clear that by (1.5) one can verify that \(a(x, s, 0)=0\) for almost every \(x \in \Omega \text{ and } \text{ for } \text{ all } s \in {\mathbb {R}}.\)

Thus, from (4.10) we have that

$$\begin{aligned} \left( a\left( x, T_{k}\left( u_{n}\right) , \nabla T_{k}\left( u_{n}\right) \right) \right) _{n} \text{ is } \text{ bounded } \text{ in } \left( L_{\varphi }(\Omega )\right) ^{N} \text{ for } \text{ all } k \ge 0 . \end{aligned}$$

Therefore, there exist a subsequence still indexed by n and a function \(l_{k}\) in \(\left( L_{\varphi }(\Omega )\right) ^{N}\) such that

$$\begin{aligned} a\left( x, T_{k}\left( u_{n}\right) , \nabla T_{k}\left( u_{n}\right) \right) \rightharpoonup l_{k} \text{ weakly } \text{ in } \left( L_{\varphi }(\Omega )\right) ^{N} \text{ for } \sigma \left( \Pi L_{\psi }, \Pi E_{\varphi }\right) . \end{aligned}$$

Then, by using the fact that \(\nabla T_{k}\left( v_{j}\right) \chi _{\Omega \backslash \Omega _{j}^{s}} \in \left( E_{\varphi }(\Omega )\right) ^{N},\) we get

$$\begin{aligned}&\int _{\Omega | \Omega _{j}^{s}} a\left( x, T_{k}\left( u_{n}\right) , \nabla T_{k}\left( u_{n}\right) \right) \cdot \nabla T_{k}\left( v_{j}\right) \phi ^{\prime }\left( \theta _{n}^{j}\right) d x\\&\quad \rightarrow \int _{\Omega \backslash \Omega _{j}^{s}} l_{k} \cdot \nabla T_{k}\left( v_{j}\right) \phi ^{\prime }\left( \theta ^{j}\right) d x \quad \text{ as } n \rightarrow \infty . \end{aligned}$$

The modular convergence of \(\left\{ v_{j}\right\} \) give

$$\begin{aligned} -\int _{\Omega \backslash \Omega _{j}} l_{k} \cdot \nabla T_{k}\left( v_{j}\right) \phi ^{\prime }\left( \theta ^{j}\right) d x \rightarrow -\int _{\Omega | \Omega ^{s}} l_{k} \cdot \nabla T_{k}(u) d x \quad \text{ as } j \rightarrow \infty \end{aligned}$$

Consequently

$$\begin{aligned} -\int _{\Omega \backslash \Omega _{j}^{*=s}} a\left( x, T_{k}\left( u_{n}\right) , \nabla T_{k}\left( u_{n}\right) \right) \cdot \nabla T_{k}\left( v_{j}\right) \phi ^{\prime }\left( \theta _{n}^{j}\right) d x=-\int _{\Omega \backslash \Omega ^{s}} l_{k} \cdot \nabla T_{k}(u) d x+\epsilon _{4}(n, j) \end{aligned}$$
(4.17)

For the fourth term, we remark that \(\psi _{m}\left( u_{n}\right) =0\) on the subset \(\left\{ \left| u_{n}\right| \ge m+1\right\} ,\) then we obtain

$$\begin{aligned}&-\int _{\left\{ \left| u_{n}\right|> k\right\} } a\left( x, u_{n}, \nabla u_{n}\right) \cdot \nabla T_{k}\left( v_{j}\right) \phi ^{\prime }\left( \theta _{n}^{j}\right) \psi _{m}\left( u_{n}\right) d x\\&\quad =-\int _{\left\{ \left| u_{n}\right| > k\right\} } a\left( x, T_{m+1}\left( u_{n}\right) , \nabla T_{m+1}\left( u_{n}\right) \right) \cdot \nabla T_{k}\left( v_{j}\right) \phi ^{\prime }\left( \theta _{n}^{j}\right) \psi _{m}\left( u_{n}\right) d x \end{aligned}$$

By using the same procedure as above we have

$$\begin{aligned}&-\int _{\left\{ \left| u_{n}\right|> k \right\} } a\left( x, T_{m+1}\left( u_{n}\right) , \nabla T_{m+1}\left( u_{n}\right) \right) \cdot \nabla T_{k}\left( v_{j}\right) \phi ^{\prime }\left( \theta _{n}^{j}\right) \psi _{m}\left( u_{n}\right) d x\\&\quad =-\int _{\{|u|>k\}} l_{m+1} \cdot \nabla T_{k}(u) \psi _{m}(u) d x+\epsilon _{5}(n, j) \end{aligned}$$

By observing that \(\nabla T_{k}(u)=0\) on the subset \(\{|u|>k\},\) we can write

$$\begin{aligned} -\int _{\left\{ \left| u_{n}\right| >k\right\} } a\left( x, u_{n}, \nabla u_{n}\right) \cdot \nabla T_{k}\left( v_{j}\right) \phi ^{\prime }\left( \theta _{n}^{j}\right) \psi _{m}\left( u_{n}\right) d x=\epsilon _{5}(n, j) \end{aligned}$$
(4.18)

For the last term of (4.15) we obtain

$$\begin{aligned} \begin{aligned}&\left| \int _{\Omega } a\left( x, u_{n}, \nabla u_{n}\right) \cdot \nabla u_{n} \phi \left( \theta _{n}^{j}\right) \psi _{m}^{\prime }\left( u_{n}\right) d x\right| \\&\quad =\left| \int _{\left\{ m \le \left| u_{k}\right| \le m+1\right\} } a\left( x, u_{n}, \nabla u_{n}\right) \cdot \nabla u_{n} \phi \left( \theta _{n}^{j}\right) \psi _{m}^{\prime }\left( u_{n}\right) d x\right| \\&\quad \le \phi (2 k) \int _{\left\{ m \le \left| u_{k}\right| \le m+1\right\} } a\left( x, u_{n}, \nabla u_{n}\right) \cdot \nabla u_{n} d x \end{aligned} \end{aligned}$$

By taking \(T_{1}\left( u_{n}-T_{m}\left( u_{n}\right) \right) \in W_{0}^{1} L_{\varphi }(\Omega )\) as test in \( (P_{n}) \) one has

$$\begin{aligned}&\int _{\left\{ m \le \left| u_{k}\right| \le m+1\right\} } a\left( x, u_{n}, \nabla u_{n}\right) \cdot \nabla u_{n} d x+\int _{\left\{ m \le \left| u_{n}\right| \le m+1\right\} } \Phi _{n}\left( u_{n}\right) \cdot \nabla u_{n} d x\\&\quad +\int _{\left\{ \left| u_{x}\right| \ge m\right\} } g_{n}\left( x, u_{n}, \nabla u_{n}\right) T_{1}\left( u_{n}-T_{m}\left( u_{n}\right) \right) d x=\left\langle f, T_{1}\left( u_{n}-T_{m}\left( u_{n}\right) \right) \right\rangle . \end{aligned}$$

by according to Lemma 3.9, we get

$$\begin{aligned} \int _{\left\{ m \le \left| u_{k}\right| \le m+1\right\} } \Phi _{n}\left( u_{n}\right) \cdot \nabla u_{n} d x=0 \end{aligned}$$

Since \(g_{n}\left( x, u_{n}, \nabla u_{n}\right) T_{1}\left( u_{n}-T_{m}\left( u_{n}\right) \right) \ge 0\) on the subset \(\left\{ \left| u_{n}\right| \ge m\right\} ,\) we have

$$\begin{aligned} \int _{\left\{ m \le \left| u_{k}\right| \le m+1\right\} } a\left( x, u_{n}, \nabla u_{n}\right) \cdot \nabla u_{n} d x \le \left\langle f, T_{1}\left( u_{n}-T_{m}\left( u_{n}\right) \right) \right\rangle \end{aligned}$$

By observing f as \(f=-{\text {div}} F,\) where \(F \in \left( E_{\varphi }(\Omega )\right) ^{N},\) and applying Young’s inequality, we get

$$\begin{aligned} \int _{\left\{ m \le \left| u_{k}\right| \le m+1\right\} } a\left( x, u_{n}, \nabla u_{n}\right) \cdot \nabla u_{n} d x \le \alpha \quad \int _{\{m \le \left| u_{n}\right| \le m+1\}} \psi \left( x,\frac{2}{\alpha }|F|\right) d x \end{aligned}$$

which implies that

$$\begin{aligned} \left| \int _{\Omega } a\left( x, u_{n}, \nabla u_{n}\right) \cdot \nabla u_{n} \phi \left( \theta _{n}^{j}\right) \psi _{m}^{\prime }\left( u_{n}\right) d x\right| \le \alpha \phi (2 k) \quad \int _{\left\{ m \le \left| u_{k}\right| \le m+1\right\} } \psi \left( x,\frac{2}{\alpha }|F|\right) d x \end{aligned}$$
(4.19)

thinks to (4.15), (4.17), (4.18) and 4.19 we get

$$\begin{aligned}&\int _{\Omega } a\left( x, u_{n}, \nabla u_{n}\right) \cdot \nabla z_{n, m}^{j} d x \nonumber \\&\quad \ge \int _{\Omega } \left( a\left( x, T_{k}\left( u_{n}\right) , \nabla T_{k}\left( u_{n}\right) \right) -a\left( x, T_{k}\left( u_{n}\right) , \nabla T_{k}\left( v_{j}\right) x_{j}^{s}\right) \right) \left( \nabla T_{k}\left( u_{n}\right) \right. \nonumber \\&\qquad \left. -\nabla T_{k}\left( v_{j}\right) x_{j}^{s}\right) \phi ^{\prime }\left( \theta _{n}^{j}\right) d x \end{aligned}$$
(4.20)
$$\begin{aligned}&\qquad -\alpha \phi (2 k) \quad \int _{\left\{ m \le \left| u_{k}\right| \le m+1\right\} } \psi \left( x,\frac{2}{\alpha }|F|\right) d x\nonumber \\&\qquad -\int _{\Omega \backslash \Omega ^{s}} l_{k} \cdot \nabla T_{k}(u) d x+e_{6}(n, j) \end{aligned}$$
(4.21)

Now, we turn to the second term on the left-hand side of (4.15) and by using the hypothesis (1.6) one has

$$\begin{aligned} \begin{aligned}&\left| \int _{\left\{ \left| u_{n}\right| \le k\right\} } g_{n}\left( x, u_{n}, \nabla u_{n}\right) \phi \left( \theta _{n}^{j}\right) d x\right| \\&\quad =\left| \int _{\left\{ \left| u_{x}\right| \le k\right\} } g_{n}\left( x, T_{k}\left( u_{n}\right) , \nabla T_{k}\left( u_{n}\right) \right) \phi \left( \theta _{n}^{j}\right) d x\right| \\&\quad \le b(k) \int _{\Omega } \varphi \left( x,\left| \nabla T_{k}\left( u_{n}\right) \right| \right) \left| \phi \left( \theta _{n}^{j}\right) \right| d x+b(k) \int _{\Omega } c(x)\left| \phi \left( \theta _{n}^{j}\right) \right| d x \\&\quad \le \frac{b(k)}{\alpha } \int _{\Omega } a_{n}\left( x, T_{k}\left( u_{n}\right) , \nabla T_{k}\left( u_{n}\right) \right) \cdot \nabla T_{k}\left( u_{n}\right) \left| \phi \left( \theta _{n}^{j}\right) \right| d x+\epsilon _{7}(n, j) \end{aligned} \end{aligned}$$

Therefore,

$$\begin{aligned}&\left| \int _{\left. \left| u_{k}\right| \le k\right\} } g_{n}\left( x, u_{n}, \nabla u_{n}\right) \phi \left( \theta _{n}^{j}\right) d x\right| \nonumber \\&\quad \le \frac{b(k)}{\alpha } \int _{\Omega }\left( a\left( x, T_{k}\left( u_{n}\right) , \nabla T_{k}\left( u_{n}\right) \right) -a\left( x, T_{k}\left( u_{n}\right) , \nabla T_{k}\left( v_{j}\right) \chi _{j}^{s}\right) \right) \left( \nabla T_{k}\left( u_{n}\right) \right. \nonumber \\&\qquad \left. -\nabla T_{k}\left( v_{j}\right) \chi _{j}^{s}\right) \left| \phi \left( \theta _{n}^{j}\right) \right| d x\nonumber \\&\qquad +\frac{b(k)}{\alpha } \int _{\Omega } a\left( x, T_{k}\left( u_{n}\right) , \nabla T_{k}\left( v_{j}\right) \chi _{j}^{s}\right) \cdot \left( \nabla T_{k}\left( u_{n}\right) -\nabla T_{k}\left( v_{j}\right) \chi _{j}^{s}\right) \left| \phi \left( \theta _{n}^{j}\right) \right| d x\nonumber \\&\qquad +\frac{b(k)}{\alpha } \int _{\Omega } a_{n}\left( x, T_{k}\left( u_{n}\right) , \nabla T_{k}\left( u_{n}\right) \right) \cdot \nabla T_{k}\left( v_{j}\right) \chi _{j}^{s}\left| \phi \left( \theta _{n}^{j}\right) \right| d x+\epsilon _{7}(n, j) \end{aligned}$$
(4.22)

Using the same procedure as above we get

$$\begin{aligned} \frac{b(k)}{\alpha } \int _{\Omega } a\left( x, T_{k}\left( u_{n}\right) , \nabla T_{k}\left( v_{j}\right) \chi _{j}^{s}\right) \cdot \left( \nabla T_{k}\left( u_{n}\right) -\nabla T_{k}\left( v_{j}\right) \chi _{j}^{s}\right) \left| \phi \left( \theta _{n}^{j}\right) \right| d x=\epsilon _{8}(n, j) \end{aligned}$$

and

$$\begin{aligned} \frac{b(k)}{\alpha } \int _{\Omega } a_{n}\left( x, T_{k}\left( u_{n}\right) , \nabla T_{k}\left( u_{n}\right) \right) \cdot \nabla T_{k}\left( v_{j}\right) \chi _{j}^{s}\left| \phi \left( \theta _{n}^{j}\right) \right| d x=\epsilon _{9}(n, j). \end{aligned}$$

thus, we obtain

$$\begin{aligned} \begin{aligned}&\left| \int _{\left\{ \left| u_{n}\right| \le k\right\} } g_{n}\left( x, u_{n}, \nabla u_{n}\right) \phi \left( \theta _{n}^{j}\right) d x\right| \le \frac{b(k)}{\alpha } \int \left( a\left( x, T_{k}\left( u_{n}\right) , \nabla T_{k}\left( u_{n}\right) \right) \right. \\&\quad \left. -a\left( x, T_{k}\left( u_{n}\right) , \nabla T_{k}\left( v_{j}\right) \chi _{j}^{s}\right) \right) \\&\quad \times \left( \nabla T_{k}\left( u_{n}\right) -\nabla T_{k}\left( v_{j}\right) \chi _{j}^{s}\right) \left| \phi \left( \theta _{n}^{j}\right) \right| d x+\epsilon _{10}(n, j) \end{aligned} \end{aligned}$$
(4.23)

By combining (4.14),(4.20) and (4.23) we have

$$\begin{aligned} \begin{aligned}&\int _{\Omega }\left( a\left( x, T_{k}\left( u_{n}\right) , \nabla T_{k}\left( u_{n}\right) \right) -a\left( x, T_{k}\left( u_{n}\right) , \nabla T_{k}\left( v_{j}\right) \chi _{j}^{s}\right) \right) \left( \nabla T_{k}\left( u_{n}\right) -\nabla T_{k}\left( v_{j}\right) \chi _{j}^{s}\right) \\&\qquad \times \left( \phi ^{\prime }\left( \theta _{n}^{j}\right) -\frac{b(k)}{\alpha } \left| \phi \left( \theta _{n}^{j}\right) \right| \right) d x\\&\quad \le \int _{\Omega \backslash \Omega ^{s}} l_{k} \cdot \nabla T_{k}(u) d x+\alpha \phi (2 k) \psi \left( x,\frac{2}{\alpha }|F|\right) d x+\epsilon _{11}(n, j) \end{aligned} \end{aligned}$$

thinks to (4.12), we get

$$\begin{aligned}&\int _{\Omega }\left( a\left( x, T_{k}\left( u_{n}\right) , \nabla T_{k}\left( u_{n}\right) \right) -a\left( x, T_{k}\left( u_{n}\right) , \nabla T_{k}\left( v_{j}\right) \chi _{j}^{s}\right) \right) \nonumber \\&\qquad \left( \nabla T_{k}\left( u_{n}\right) -\nabla T_{k}\left( v_{j}\right) \chi _{j}^{s}\right) \rho (x) d x\nonumber \\&\quad \le 2 \int _{\Omega \backslash \Omega ^{s}} l_{k} \cdot \nabla T_{k}(u) d x+2 \alpha \phi (2 k) \nonumber \\&\qquad \int _{\left\{ m \le \left| u_{n}\right| \le m+1\right\} } \psi \left( x,\frac{2}{\alpha }|F|\right) d x+\epsilon _{11}(n, j) \end{aligned}$$
(4.24)

On the other hand, we have

$$\begin{aligned}&\int _{\Omega }\left( a\left( x, T_{k}\left( u_{n}\right) , \nabla T_{k}\left( u_{n}\right) \right) -a\left( x, T_{k}\left( u_{n}\right) , \nabla T_{k}(u) \chi ^{s}\right) \right) \left( \nabla T_{k}\left( u_{n}\right) -\nabla T_{k}(u) \chi ^{s}\right) d x\\&\quad =\int _{\Omega }\left( a\left( x, T_{k}\left( u_{n}\right) , \nabla T_{k}\left( u_{n}\right) \right) -a\left( x, T_{k}\left( u_{n}\right) , \nabla T_{k}\left( v_{j}\right) \chi _{j}^{s}\right) \right) \\&\qquad \left( \nabla T_{k}\left( u_{n}\right) -\nabla T_{k}\left( v_{j}\right) \chi _{j}^{s}\right) d x\\&\qquad +\int _{\Omega } a\left( x, T_{k}\left( u_{n}\right) , \nabla T_{k}\left( u_{n}\right) \right) \cdot \left( \nabla T_{k}\left( v_{j}\right) \chi _{j}^{s}-\nabla T_{k}(u) \chi ^{s}\right) d x\nonumber \\&\qquad -\int _{\Omega } a\left( x, T_{k}\left( u_{n}\right) , \nabla T_{k}(u) \chi ^{s}\right) \cdot \left( \nabla T_{k}\left( u_{n}\right) -\nabla T_{k}(u) \chi ^{s}\right) d x \nonumber \\&\qquad +\int _{\Omega } a\left( x, T_{k}\left( u_{n}\right) , \nabla T_{k}\left( v_{j}\right) \chi _{j}^{s}\right) \cdot \left( \nabla T_{k}\left( u_{n}\right) -\nabla T_{k}\left( v_{j}\right) \chi _{j}^{s}\right) d x \end{aligned}$$

We will passe to the limit in n and then in j in the last three terms of the right-hand side of the above equality.

using the same procedure as is done in (4.15) and (4.22), we get

$$\begin{aligned} \begin{aligned}&\int _{\Omega } a\left( x, T_{k}\left( u_{n}\right) , \nabla T_{k}\left( u_{n}\right) \right) \cdot \left( \nabla T_{k}\left( v_{j}\right) \chi _{j}^{s}-\nabla T_{k}(u) \chi ^{s}\right) d x=\epsilon _{12}(n, j)\\&\int _{\Omega } a\left( x, T_{k}\left( u_{n}\right) , \nabla T_{k}(u) \chi ^{s}\right) \cdot \left( \nabla T_{k}\left( u_{n}\right) -\nabla T_{k}(u) \chi ^{s}\right) d x=\epsilon _{13}(n, j)\\&\int _{\Omega } a\left( x, T_{k}\left( u_{n}\right) , \nabla T_{k}\left( v_{j}\right) \chi _{j}^{s}\right) \cdot \left( \nabla T_{k}\left( u_{n}\right) -\nabla T_{k}\left( v_{j}\right) \chi _{j}^{s}\right) d x=\epsilon _{14}(n, j) \end{aligned} \end{aligned}$$
(4.25)

Therefore,

$$\begin{aligned}&\begin{array}{l} \displaystyle {\int _{\Omega }\left( a\left( x, T_{k}\left( u_{n}\right) , \nabla T_{k}\left( u_{n}\right) \right) -a\left( x, T_{k}\left( u_{n}\right) , \nabla T_{k}(u) \chi ^{s}\right) \right) \left( \nabla T_{k}\left( u_{n}\right) -\nabla T_{k}(u) \chi ^{s}\right) d x}\end{array}\nonumber \\&\quad =\int _{\Omega }\left( a\left( x, T_{k}\left( u_{n}\right) , \nabla T_{k}\left( u_{n}\right) \right) -a\left( x, T_{k}\left( u_{n}\right) , \nabla T_{k}\left( v_{j}\right) \chi _{j}^{s}\right) \right) \nonumber \\&\qquad \left( \nabla T_{k}\left( u_{n}\right) -\nabla T_{k}\left( v_{j}\right) \chi _{j}^{s}\right) d x+\epsilon _{15}(n, j) \end{aligned}$$
(4.26)

Let \(r \le s\). Thinks to (1.4) , (4.24) and (4.26) we have

$$\begin{aligned} \begin{aligned} 0&\le \int _{\Omega ^{r}}\left( a\left( x, T_{k}\left( u_{n}\right) , \nabla T_{k}\left( u_{n}\right) \right) -a\left( x, T_{k}\left( u_{n}\right) , \nabla T_{k}(u)\right) \right) \left( \nabla T_{k}\left( u_{n}\right) -\nabla T_{k}(u)\right) d x \\&\le \int _{\Omega ^{s}}\left( a\left( x, T_{k}\left( u_{n}\right) , \nabla T_{k}\left( u_{n}\right) \right) -a\left( x, T_{k}\left( u_{n}\right) , \nabla T_{k}(u)\right) \right) \left( \nabla T_{k}\left( u_{n}\right) -\nabla T_{k}(u)\right) d x \\&=\int _{\Omega ^{s}}\left( a\left( x, T_{k}\left( u_{n}\right) , \nabla T_{k}\left( u_{n}\right) \right) -a\left( x, T_{k}\left( u_{n}\right) , \nabla T_{k}(u) \chi ^{s}\right) \right) \left( \nabla T_{k}\left( u_{n}\right) -\nabla T_{k}(u) \chi ^{s}\right) d x \\&\le \int _{\Omega }\left( a\left( x, T_{k}\left( u_{n}\right) , \nabla T_{k}\left( u_{n}\right) \right) -a\left( x, T_{k}\left( u_{n}\right) , \nabla T_{k}(u) \chi ^{s}\right) \right) \left( \nabla T_{k}\left( u_{n}\right) -\nabla T_{k}(u) \chi ^{s}\right) d x \\&=\int _{\Omega }\left( a\left( x, T_{k}\left( u_{n}\right) , \nabla T_{k}\left( u_{n}\right) \right) -a\left( x, T_{k}\left( u_{n}\right) , \nabla T_{k}\left( v_{j}\right) \chi _{j}^{s}\right) \right) \\&\quad \left( \nabla T_{k}\left( u_{n}\right) -\nabla T_{k}\left( v_{j}\right) \chi _{j}^{s}\right) d x+\epsilon _{15}(n, j) \end{aligned} \end{aligned}$$

by passing to the limit in n and then in j one has,

$$\begin{aligned} \begin{aligned} 0&\le \underset{n \rightarrow \infty }{\lim \sup } \int \left( a\left( x, T_{k}\left( u_{n}\right) , \nabla T_{k}\left( u_{n}\right) \right) -a\left( x, T_{k}\left( u_{n}\right) , \nabla T_{k}(u)\right) \right) \left( \nabla T_{k}\left( u_{n}\right) -\nabla T_{k}(u)\right) d x \\&\le 2 \int _{\Omega \backslash \Omega ^{s}} l_{k} \cdot \nabla T_{k}(u) d x+2 \alpha \phi (2 k) \int _{\{m \le |u| \le m+1\}} \psi \left( x,\frac{2}{\alpha }|F|\right) d x.\end{aligned} \end{aligned}$$

Let \(s \rightarrow +\infty \) and \(m \rightarrow +\infty ,\) using the fact that \(l_{k} \cdot \nabla T_{k}(u) \in L^{1}(\Omega ),|F| \in \left( E_{\varphi }(\Omega )\right) ^{N},\left| \Omega \backslash \Omega ^{s}\right| \rightarrow 0\) and \(|\{m \le |u| \le m+1\}| \rightarrow 0,\) we obtain

$$\begin{aligned}&\int _{\Omega ^{r}}\left( a\left( x, T_{k}\left( u_{n}\right) , \nabla T_{k}\left( u_{n}\right) \right) -a\left( x, T_{k}\left( u_{n}\right) , \nabla T_{k}(u)\right) \right) \\&\quad \cdot \left( \nabla T_{k}\left( u_{n}\right) -\nabla T_{k}(u)\right) d x \rightarrow 0 \quad \text{ as } n \rightarrow \infty \end{aligned}$$

Thinks to [31] there exists a subsequence of \(\left\{ u_{n}\right\} \) still indexed by n such that

$$\begin{aligned} \nabla u_{n} \rightarrow \nabla u \quad \text{ a.e.in } \Omega \end{aligned}$$
(4.27)

Thus, by taking account that (4.7),(4.9) and (4.10) we can apply [33], Theorem 14.6] to obtain \(a(x, u, \nabla u) \in \left( L_{\varphi }(\Omega )\right) ^{N}\) and

$$\begin{aligned} a\left( x, u_{n}, \nabla u_{n}\right) \rightharpoonup a(x, u, \nabla u) \text{ weakly } \text{ in } \left( L_{\varphi }(\Omega )\right) ^{N} \text{ for } \sigma \left( \Pi L_{\varphi }(\Omega ), \Pi E_{\psi }(\Omega )\right) . \end{aligned}$$
(4.28)

4.1.5.

Now,we shall prove that

$$\begin{aligned} \text{ for } \text{ every } k>0,\quad T_{k}\left( u_{n}\right) \rightarrow T_{k}(u) \quad \text{ in } W_{0}^{1} L_{\varphi }(\Omega ) \text{ for } \text{ the } \text{ modular } \text{ convegence } \end{aligned}$$

.

From inequality (4.24), we obtain

$$\begin{aligned}&\int _{\Omega } a\left( x, T_{k}\left( u_{n}\right) , \nabla T_{k}\left( u_{n}\right) \right) \cdot \nabla T_{k}\left( u_{n}\right) d x \le \int _{\Omega } a\left( x, T_{k}\left( u_{n}\right) , \nabla T_{k}\left( u_{n}\right) \right) \cdot \nabla T_{k}\left( v_{j}\right) \chi _{j}^{s} d x\\&\quad +\int _{\Omega } a\left( x, T_{k}\left( u_{n}\right) , \nabla T_{k}\left( v_{j}\right) x_{j}^{s}\right) \cdot \left( \nabla T_{k}\left( u_{n}\right) -\nabla T_{k}\left( v_{j}\right) \chi _{j}^{s}\right) d x \\&\quad +2 \alpha \phi (2 k) \quad \int _{\left\{ m \le | u_{k} | \le m+1\right\} } \psi \left( x,\frac{2}{\alpha }|F|\right) d x\\&\quad +2 \int _{\Omega | \Omega ^{s}} a\left( x, T_{k}(u), \nabla T_{k}(u)\right) \cdot \nabla T_{k}(u) d x+\epsilon _{11}(n, j). \end{aligned}$$

thinks to (4.25), we obtain

$$\begin{aligned}&\int _{\Omega } a\left( x, T_{k}\left( u_{n}\right) , \nabla T_{k}\left( u_{n}\right) \right) \cdot \nabla T_{k}\left( u_{n}\right) d x\\&\quad \le \int _{\Omega } a\left( x, T_{k}\left( u_{n}\right) , \nabla T_{k}\left( u_{n}\right) \right) \cdot \nabla T_{k}\left( v_{j}\right) \chi _{j}^{s} d x+2 \alpha \phi (2 k)\\&\qquad \int _{\left\{ m \le \left| u_{n}\right| \le m+1\right\} } \psi \left( x,\frac{2}{\alpha }|F|\right) d x\\&\qquad +2 \int _{\Omega | \Omega ^{s}} a\left( x, T_{k}(u), \nabla T_{k}(u)\right) \cdot \nabla T_{k}(u) d x+\epsilon _{17}(n, j). \end{aligned}$$

the passage to the limitto the limit in n on both sides of this inequality and using (4.28) implies that

$$\begin{aligned}&\limsup _{n \rightarrow \infty } \int _{\Omega } a\left( x, T_{k}\left( u_{n}\right) , \nabla T_{k}\left( u_{n}\right) \right) \cdot \nabla T_{k}\left( u_{n}\right) d x\\&\quad \le \int _{\Omega } a\left( x, T_{k}(u), \nabla T_{k}(u)\right) \cdot \nabla T_{k}\left( v_{j}\right) \chi _{j}^{s} d x+2 \alpha \phi (2 k)\\&\qquad \int _{\{m \le |u| \le m+1\}} \psi \left( x,\frac{2}{\alpha }|F|\right) d x\\&\qquad +2 \int _{\Omega \backslash \Omega ^{s}} a\left( x, T_{k}(u), \nabla T_{k}(u)\right) \cdot \nabla T_{k}(u) d x. \end{aligned}$$

and by passing to the limit in j we obtain

$$\begin{aligned}&\limsup _{n \rightarrow \infty } \int _{\Omega } a\left( x, T_{k}\left( u_{n}\right) , \nabla T_{k}\left( u_{n}\right) \right) \cdot \nabla T_{k}\left( u_{n}\right) d x\\&\quad \le \int _{\Omega } a\left( x, T_{k}(u), \nabla T_{k}(u)\right) \cdot \nabla T_{k}(u) \chi ^{s} d x+ 2 \alpha \phi (2 k)\\&\qquad \int _{\{m \le |u| \le m+1\}} \psi \left( x,\frac{2}{\alpha }|F|\right) d x\\&\qquad +2 \int _{\Omega \backslash \Omega ^{s}} a\left( x, T_{k}(u), \nabla T_{k}(u)\right) \cdot \nabla T_{k}(u) d x. \end{aligned}$$

Let s and \(m \rightarrow \infty ,\) we get

$$\begin{aligned} \displaystyle {\limsup _{n \rightarrow \infty } \int _{\Omega } a\left( x, T_{k}\left( u_{n}\right) , \nabla T_{k}\left( u_{n}\right) \right) \cdot \nabla T_{k}\left( u_{n}\right) d x \le \int _{\Omega } a\left( x, T_{k}(u), \nabla T_{k}(u)\right) \cdot \nabla T_{k}(u) d x} \end{aligned}$$

Now, thinks to (1.5),(4.4),(4.27) and applying Fatou’s lemma, we get

$$\begin{aligned} \int _{\Omega } a\left( x, T_{k}(u), \nabla T_{k}(u)\right) \cdot \nabla T_{k}(u) d x \le \liminf _{n \rightarrow \infty } \int _{\Omega } a\left( x, T_{k}\left( u_{n}\right) , \nabla T_{k}\left( u_{n}\right) \right) \cdot \nabla T_{k}\left( u_{n}\right) d x \end{aligned}$$

thus,

$$\begin{aligned} \lim _{n \rightarrow \infty } \int _{\Omega } a\left( x, T_{k}\left( u_{n}\right) , \nabla T_{k}\left( u_{n}\right) \right) \cdot \nabla T_{k}\left( u_{n}\right) d x=\int _{\Omega } a\left( x, T_{k}(u), \nabla T_{k}(u)\right) \cdot \nabla T_{k}(u) d x \end{aligned}$$

In view of Lemma 3.5, we deduce that for every \(k>0\)

$$\begin{aligned} a\left( x, T_{k}\left( u_{n}\right) , \nabla T_{k}\left( u_{n}\right) \right) \cdot \nabla T_{k}\left( u_{n}\right) \rightarrow a\left( x, T_{k}(u), \nabla T_{k}(u)\right) \cdot \nabla T_{k}(u) \quad \text{ strongly } \text{ in } L^{1}(\Omega ) \end{aligned}$$
(4.29)

by vertu of hypothesis (1.5) and using the convexity of \(\varphi \) we get

$$\begin{aligned}&\varphi \left( x,\frac{\left| \nabla T_{k}\left( u_{n}\right) -\nabla T_{k}(u)\right| }{2}\right) \\&\quad \le \frac{1}{2 \alpha } a\left( x, T_{k}\left( u_{n}\right) , \nabla T_{k}\left( u_{n}\right) \right) \cdot \nabla T_{k}\left( u_{n}\right) +\frac{1}{2 \alpha } a\left( x, T_{k}(u), \nabla T_{k}(u)\right) \cdot \nabla T_{k}(u) \end{aligned}$$

By applying Vitali’s theorem we obtain

$$\begin{aligned} \lim _{|\varepsilon | \rightarrow 0} \sup _{n} \int _{E}\varphi \left( x,\frac{\left| \nabla T_{k}\left( u_{n}\right) -\nabla T_{k}(u)\right| }{2}\right) d x=0 \end{aligned}$$

Consequently, for every \(k>0\)

$$\begin{aligned} T_{k}\left( u_{n}\right) \rightarrow T_{k}(u) \quad \text{ in } W_{0}^{1} L_{\varphi }(\Omega ). \end{aligned}$$

for the modular convegence.

4.1.6.

We shall show that

$$\begin{aligned} g_{n}\left( x, u_{n}, \nabla u_{n}\right) \rightarrow g(x, u, \nabla u) \quad in \quad L^{1}(\Omega ) \end{aligned}$$
(4.30)

From (4.9) and (4.27) we have

$$\begin{aligned} g_{n}\left( x, u_{n}, \nabla u_{n}\right) \rightarrow g(x, u, \nabla u) \quad \text{ a.e. } \text{ in } \Omega \end{aligned}$$
(4.31)

Let E be a measurable subset of \(\Omega \) and let \(m>0 .\) by taking account of (1.5) and (1.6) we obtain

$$\begin{aligned}&\int _{E}\left| g_{n}\left( x, u_{n}, \nabla u_{n}\right) \right| d x=\int _{E \cap \left[ \left| u_{k}\right| \le m\right\} }\left| g_{n}\left( x, u_{n}, \nabla u_{n}\right) \right| d x\\&\qquad +\int _{E \cap \left\{ \left| u_{n}\right| >m\right\} }\left| g_{n}\left( x, u_{n}, \nabla u_{n}\right) \right| d x\\&\quad \le b(m) \int _{E} c(x) d x+b(m) \int _{E} a\left( x, T_{m}\left( u_{n}\right) , \nabla T_{m}\left( u_{n}\right) \right) \cdot \nabla T_{m}\left( u_{n}\right) d x\\&\qquad +\frac{1}{m} \int _{\Omega } g_{n}\left( x, u_{n}, \nabla u_{n}\right) u_{n} d x \end{aligned}$$

By (1.7) and (4.6) it follows that

$$\begin{aligned} \lim _{m \rightarrow \infty } \frac{1}{m} \int _{\Omega } g_{n}\left( x, u_{n}, \nabla u_{n}\right) u_{n} d x=0 \end{aligned}$$

By using (4.29) the sequence

$$\begin{aligned} \left\{ a\left( x, T_{m}\left( u_{n}\right) , \nabla T_{m}\left( u_{n}\right) \right) \cdot \nabla T_{m}\left( u_{n}\right) \right\} _{n} \text{ is } \text{ equi-integrable } , \end{aligned}$$

Consequently

$$\begin{aligned} \lim _{|E| \rightarrow 0} \sup _{n} \int _{E} a\left( x, T_{m}\left( u_{n}\right) , \nabla T_{m}\left( u_{n}\right) \right) \cdot \nabla T_{m}\left( u_{n}\right) d x=0 \end{aligned}$$

This proves that \(g_{n}\left( x, u_{n}, \nabla u_{n}\right) \) is equi-integrable.

Therefore, Vitali’s theorem allows us to get

$$\begin{aligned} g(x, u, \nabla u) \in L^{1}(\Omega ), \end{aligned}$$

and

$$\begin{aligned} g_{n}\left( x, u_{n}, \nabla u_{n}\right) \rightarrow g(x, u, \nabla u) \quad \text{ strongly } \text{ in } L^{1}(\Omega ). \end{aligned}$$
(4.32)

4.1.7.

In this subsubsection we prove that

$$\begin{aligned} \lim _{m \rightarrow \infty } \int _{\{m \le |u| \le m+1\}} a(x, u, \nabla u) \cdot \nabla u d x=0 \end{aligned}$$
(4.33)

for any \( m \ge 0 \) we have

$$\begin{aligned}&\int _{\left\{ m \le \left| u_{n}\right| \le m+1\right\} } a\left( x, u_{n}, \nabla u_{n}\right) \cdot \nabla u_{n} d x =\int _{\Omega } a\left( x, u_{n}, \nabla u_{n}\right) \cdot \left( \nabla T_{m+1}\left( u_{n}\right) -\nabla T_{m}\left( u_{n}\right) \right) d x\\&\quad =\int _{\Omega } a\left( x, T_{m+1}\left( u_{n}\right) , \nabla T_{m+1}\left( u_{n}\right) \right) \cdot \nabla T_{m+1}\left( u_{n}\right) d x\\&\qquad -\int _{\Omega } a\left( x, T_{m}\left( u_{n}\right) , \nabla T_{m}\left( u_{n}\right) \right) \cdot \nabla T_{m}\left( u_{n}\right) d x \end{aligned}$$

Thinks to (4.29) and passing to the limit as \(n \rightarrow +\infty \) for fixed \(m \ge 0\)

$$\begin{aligned}&\lim _{n \rightarrow \infty } \int _{\left\{ m \le \left| u_{n}\right| \le m+1\right\} } a\left( x, u_{n}, \nabla u_{n}\right) \cdot \nabla u_{n} d x \\&\quad =\int _{\Omega } a\left( x, T_{m+1}(u), \nabla T_{m+1}(u)\right) \cdot \nabla T_{m+1}(u) d x-\int _{\Omega } a\left( x, T_{m}(u), \nabla T_{m}(u)\right) \nabla T_{m}(u) d x\\&\quad =\int _{\Omega } a(x, u, \nabla u) \cdot \left( \nabla T_{m+1}(u)-\nabla T_{m}(u)\right) d x\\&\quad =\int _{\{m \le |u| \le m+1\}}a(x, u, \nabla u) \cdot \nabla u d x. \end{aligned}$$

according to (4.11), we can pass to the limit as \(m \rightarrow +\infty \) in order to have (4.33).

4.1.8.

Finally, in this step thanks to (4.29) and Lemma 3.5, one has

$$\begin{aligned} \displaystyle { a\left( x, u_{n}, \nabla u_{n}\right) \cdot \nabla u_{n} \rightarrow a(x, u, \nabla u) \cdot \nabla u \quad \text{ strongly } \text{ in } L^{1}(\Omega )} \end{aligned}$$
(4.34)

Let \(h \in {\mathbb {C}}_{c}^{1}({\mathbb {R}})\) and \(\varrho \in {\mathcal {D}}(\Omega ) .\) we choose \(h\left( u_{n}\right) \varrho \) as a test function in \( (P_{n}) \) we obtain

$$\begin{aligned}&\int _{\Omega } a\left( x, u_{n}, \nabla u_{n}\right) \cdot \nabla u_{n} h^{\prime }\left( u_{n}\right) \varrho d x+\int _{\Omega } \rho (x)a\left( x, u_{n}, \nabla u_{n}\right) \cdot \nabla \varrho h\left( u_{n}\right) d x\nonumber \\&\quad +\int _{\Omega } \Phi _{n}\left( u_{n}\right) \cdot \nabla \left( h\left( u_{n}\right) \varrho \right) d x+\int _{\Omega } g_{n}\left( x, u_{n}, \nabla u_{n}\right) h\left( u_{n}\right) \varrho d x=\left\langle f, h\left( u_{n}\right) \varrho \right\rangle \end{aligned}$$
(4.35)

Now, we can pass to the limit as \(n \rightarrow +\infty \) in each term of equality (4.35). since h and \(h^{\prime }\) have a compact support on \({\mathbb {R}},\) there exists a real number \(v>0,\) such that supp \(h \subset [-v, v]\) and supp \(h^{\prime } \subset [-v, v] .\) For \(n>v,\) we can have

$$\begin{aligned} \Phi _{n}(t) h(t)=\Phi \left( T_{v}(t)\right) h(t) \quad \text{ and } \quad \Phi _{n}(t) h^{\prime }(t)=\Phi \left( T_{v}(t)\right) h^{\prime }(t) \end{aligned}$$

Moreover,

$$\begin{aligned} \text{ the } \text{ functions } \Phi h \text{ and } \Phi h^{\prime } \text{ belong } \text{ to } \left( {\mathcal {C}}^{0}({\mathbb {R}}) \cap L^{\infty }({\mathbb {R}})\right) ^{N} . \end{aligned}$$

Now we can see that the sequence \(\left\{ h\left( u_{n}\right) \varrho \right\} _{n}\) is bounded in \(W_{0}^{1} L_{\varphi }(\Omega ).\)

Indeed, let \(c^{'} >0\) be a positive constant such that \(\left\| h\left( u_{n}\right) \nabla \varrho \right\| _{\infty } \le c^{'}\) and \(\left\| h^{\prime }\left( u_{n}\right) \varrho \right\| _{\infty } \le c^{'}\).

Thinks to (3.2) we obtain

$$\begin{aligned} \begin{aligned} \int _{\Omega } \varphi \left( x,\frac{\left| \nabla \left( h\left( u_{n}\right) \varrho \right) \right| }{2 c^{'}}\right) d x&\le \int _{\Omega } \varphi \left( x,\frac{\left| h\left( u_{n}\right) \nabla \varrho \right| +\left| h^{\prime }\left( u_{n}\right) \varrho \Vert \nabla u_{n}\right| }{2 c^{'}}\right) d x \\&\le \frac{1}{2} \int _{\Omega } \varphi (x, 1) d x+\frac{1}{2} \int _{\Omega } \varphi \left( x,\left| \nabla T_{M}\left( u_{n}\right) \right| \right) d x \\&\le c \end{aligned} \end{aligned}$$

Which jointly with (4.9) it follows that

$$\begin{aligned} h\left( u_{n}\right) \varrho \rightarrow h(u) \varrho \text{ weakly } \text{ in } W_{0}^{1} L_{\varphi }(\Omega ) \text{ for } \text{ for } \sigma \left( \Pi L_{\varphi }, \Pi E_{\psi }\right) . \end{aligned}$$
(4.36)

Which give

$$\begin{aligned} \left\langle f, h\left( u_{n}\right) \varphi \right\rangle \rightarrow \langle f, h(u) \varphi \rangle . \end{aligned}$$

Let E be a measurable subset of \(\Omega .\) we pose \(c_{v}=\max _{ |t| \le v} \Phi (t) .\) And denoting by \( \Vert v|_{\varphi , \Omega } \) the Orlicz norm of a function \(v \in L_{\varphi }(\Omega ) .\) We thinking to the strengthened \( H \ddot{o}lder \) inequality with both Orlicz and Luxemburg norms, we have

$$\begin{aligned}&\left\| \Phi \left( T_{v}\left( u_{n}\right) \right) \chi _{E}\right\| _{\psi ,\Omega } =\sup _{\Vert v\Vert _{\varphi ,\Omega } \le 1}\left| \int _{E} \Phi \left( T_{v}\left( u_{n}\right) \right) v d x\right| \\&\quad \le c_{v} \sup _{\Vert v\Vert _{\varphi ,\Omega } \le 1}\left\| \chi _{E}\right\| _{\psi ,\Omega }\Vert v\Vert _{\varphi ,\Omega } \\&\quad \le c_{v}|E| \varphi ^{-1}\left( x,\frac{1}{|E|}\right) \end{aligned}$$

Consequently,

$$\begin{aligned} \lim _{|E| \rightarrow 0} \sup _{n}\left\| \Phi \left( T_{v}\left( u_{n}\right) \right) \chi _{E}\right\| _{(\psi ,\Omega )}=0 \end{aligned}$$

Then, in view of (4.9) and by applying [33], Lemma 11.2], we get

$$\begin{aligned} \Phi \left( T_{v}\left( u_{n}\right) \right) \rightarrow \Phi \left( T_{v}(u)\right) \quad \text{ strongly } \text{ in } \left( E_{\psi }(\Omega )\right) ^{N} \end{aligned}$$

which together with (4.36) allow us to pass to the limit in the third term of (4.35) to obtain

$$\begin{aligned} \int _{\Omega } \Phi \left( T_{v}\left( u_{n}\right) \right) \cdot \nabla \left( h\left( u_{n}\right) \varrho \right) d x \rightarrow \int _{\Omega } \Phi \left( T_{v}(u)\right) \cdot \nabla (h(u) \varrho ) d x \end{aligned}$$

Observing that

$$\begin{aligned} \left| a\left( x, u_{n}, \nabla u_{n}\right) \cdot \nabla u_{n} h^{\prime }\left( u_{n}\right) \varrho \right| \le c^{'} a\left( x, u_{n}, \nabla u_{n}\right) \cdot \nabla u_{n} \end{aligned}$$

Therefore, thinks to (4.34) and applying Vitali’s theorem, we get

$$\begin{aligned} \int _{\Omega } a\left( x, u_{n}, \nabla u_{n}\right) \cdot \nabla u_{n} h^{\prime }\left( u_{n}\right) \varrho d x \rightarrow \int _{\Omega } a(x, u, \nabla u) \cdot \nabla u h^{\prime }(u) \varrho d x \end{aligned}$$

Concerning the second term of (4.35) using the same procedure as above we obtain

$$\begin{aligned} h\left( u_{n}\right) \nabla \varrho \rightarrow h(u) \nabla \varrho \quad \text{ strongly } \text{ in } \left( E_{\varphi }(\Omega )\right) ^{N} \end{aligned}$$

which jointly with (4.28) implies that

$$\begin{aligned} \int _{\Omega } a\left( x, u_{n}, \nabla u_{n}\right) \cdot \nabla \varrho h\left( u_{n}\right) d x \rightarrow \int _{\Omega } a(x, u, \nabla u) \cdot \nabla \varrho h(u) d x \end{aligned}$$

Remark that \(h\left( u_{n}\right) \varrho \rightarrow h(u) \varrho \) weakly in \(L^{\infty }(\Omega )\) for \(\sigma ^{*}\left( L^{\infty }, L^{1}\right) \) with (4.30) we can pass to the limit in the Fourth term of (4.35) in order to have

$$\begin{aligned} \int _{\Omega } g_{n}\left( x, u_{n}, \nabla u_{n}\right) h\left( u_{n}\right) \varrho d x \rightarrow \int _{\Omega } g(x, u, \nabla u) h(u) \varrho d x \end{aligned}$$

Finally, we can pass to the limit in each term of (4.35) so as to obtain

$$\begin{aligned}&\int _{\Omega } a(x, u, \nabla u) \cdot \left[ \nabla \varphi h(u)+h^{\prime }(u) \varrho \nabla u\right] d x+\int \Phi (u) h^{\prime }(u) \varrho \cdot \nabla u d x \\&\quad +\int _{\Omega } \Phi (u) h(u) \cdot \nabla \varrho d x+\int _{\Omega } g(x, u, \nabla u) h(u) \varrho d x =\langle f, h(u) \varrho \rangle \end{aligned}$$

for all \(h \in {\mathcal {C}}_{c}^{1}({\mathbb {R}})\) and for all \(\varrho \in {\mathcal {D}}(\Omega ) .\) Thus, as well (1.7),(4.6) and (4.31) we apply Fatou’s lemma to get \( g(x, u, \nabla u) u \in L^{1}(\Omega )\).

Consequently, thinks to (4.9),(4.28),(4.32),(4.33), the function u is a renormalized solution of problem \( ({\mathcal {P}}) \) .